Monta roolia avoinna

31.5.2019 klo 22.16, kirjoittaja
Kategoriat: Kosmokseen kirjoitettua , Kosmologia

Tällä viikolla Helsingin yliopiston fysiikan osastolla oli Higgsin kenttään keskittyvä pieni konferenssi NonMinimalHiggs. Ohjelmassa oli teoreettisia pohdintoja, kokeita lähellä olevien mallien rakentamista ja havaintojen analysointia. Higgs nivookin yhteen useita hiukkasfysiikan ja kosmologian isoja kysymyksiä.

Higgsin kentän tunnetuin rooli lienee se, että se antaa massan kaikille tunnetuille hiukkasille (paitsi ehkä neutriinoille). Kun Higgsin hiukkanen (joka on kentän tihentymä) saatiin vuonna 2012 pitkän jahdin jälkeen kiinni, massojen mekanismista annettiin seuraavana vuonna Nobelin palkinto. Tämä oli kuitenkin vain välietappi, tutkimusmatka jatkuu.

Se, että hiukkasfysiikan Standardimallin hiukkaset vuorovaikuttavat Higgsin kentän kanssa, ja Higgsin kenttiä on vain yksi, on yksinkertaisin tapa antaa hiukkasille massa. Ennen Higgsin löytämistä oli pitkään tutkittu vaihtoehtoja, missä Higgsin kenttiä on useampia, niin että hiukkaset saavat massansa eri kentiltä. Näin käy esimerkiksi supersymmetrisissä malleissa. Toisaalta oli ehdotettu, että ehkä Higgs ei ole alkeishiukkanen, vaan koostuu muista hiukkasista, kuten protoni koostuu kvarkeista.

Tällaisia ideoita tutkitaan vieläkin tarkasti, ja konferenssissa käytiin läpi useita eri mahdollisuuksia. Niin teoriapuolen pohdinnat ja tarkasteluihin käytetyt ohjelmistopakkaukset kuin datatuotteet ja analyysin menetelmät ovat vuosien aikana hioutuneet tarkoiksi, mutta toistaiseksi vuosikymmenten työn tulos helppo tiivistää: mitään poikkeamia Standardimallista ei ole löytynyt.

Yksi ajankohtainen keskustelunaihe on se, millainen on seuraava hiukkaskiihdytin, joka porautuu Higgsin löytänyttä LHC:tä syvemmälle, minne se rakennetaan, millä aikataululla ja kenen rahoilla. Konferenssissa esiteltiin sitä, miten eri kiihdyttimet luotaisivat Higgsin ja muiden hiukkasten eri ominaisuuksia. Yhtenä perusteluna seuraavan sukupolven kiihdyttimille esitettiin, että ne pystyvät luotaamaan Standardimallia prosentin tarkkuudella, minkä suuruisia poikkeamia Standardimallista nykyiset mallit juuri ennustavat.

Perustelussa näkyy asialle omistautuneiden asiantuntijoiden näkökulmaharha: nykyiset mallit ennustavat prosentin suuruisia poikkeamia vain siksi, että isommat olisi jo nähty LHC:ssä. Vastaavasti edellisen sukupolven mallit ennustivat juuri sen suuruisia poikkeamia, mitä edelliset kiihdyttimet eivät olisi nähneet, mutta jotka olivat LHC:n ulottuvilla.

Oli erinomaisen perusteltua odottaa, että LHC löytää Higgsin hiukkasen (tai jotain eksoottisempaa), mutta sitä pidemmälle ei ole takeita. Nyt liikutaan avomerellä vailla tietoa siitä, koska maata löytyy, mutta on tietysti lohdullista ajatella, että huomenna horisontissa kajastaa uusi maailma. Tämä ei tarkoita sitä, etteikö kiihdyttimiä kannattaisi rakentaa: tuntemattomilla vesillä voi tehdä odottamattomia löytöjä, mutta on syytä olla rehellinen ehdotusten perusteista.

Tässäkin konferenssissa näkyi se trendi, että kaiken teoriasta ja huipulta alas rakennetuista yhtenäisteorioista puhutaan entistä vähemmän. Enemmän keskitytään rakentamaan tunnetulta pohjalta ylöspäin, ja katsomaan mitä tunnettujen asioiden läheisyydestä voisi löytyä.

Toinen selvä trendi on kosmologian nousu. Higgs voi olla vastuussa kosmisesta inflaatiosta, joka synnytti galaksien ja kaiken muun rakenteen siemenet. Vaikka näin ei olisi, Higgsin käytökseen varhaisessa maailmankaikkeudessa liittyy kiinnostavia kysymyksiä, kuten se, onko tyhjä avaruus vakaa vai voiko maailmankaikkeus tuhoutua noin vain.

Higgsiin liittyvän kosmologian tutkiminen on hyvä esimerkki siitä, miten ideat kulkeutuvat fysiikan alueilta toisille ja pölyttävät tutkijoita. Monia Higgsin kosmologiaan liittyviä aiheita ruvettiin tutkimaan vasta 2000-luvulla Higgsin hiukkaskiihdytintutkimuksen kehityksen myötä. Kyse ei niinkään ole siitä, että kiihdyttimet olisivat kertoneet tärkeää uutta tietoa: sitäkin oli, mutta enimmäkseen kaikki tarpeellinen tiedettiin jo 1980-luvulla. Tärkeämpää oli se, että kun Higgsistä puhuttiin ja vaihdettiin ideoita, niin siihen liittyviä kysymyksiä tultiin miettineeksi tarkemmin.

Yksi kosmologinen aihe, jota on tutkittu jo pitkään, on se, meneekö aineen ja antiaineen välisen epäsymmetrian alkuperä sekin Higgsin piikkiin. Kirjoitan tästä kiehtovasta mahdollisuudesta ehkä myöhemmin tarkemmin, tässä vain pikainen katsaus. Muinaisina aikoina Higgsin kentällä ei ollut erityistä suuntaa: kuten nestemäinen vesi, se oli samanlainen kaikissa suunnissa. Kun maailmankaikkeus oli sekunnin miljardisosan ikäinen, lämpötila laski niin alhaiseksi (miljoona miljardia astetta), että Higgsin kenttä jäätyi tiettyyn asentoon. Jäätyminen alkaa eri paikoissa eri aikaan, joten jäätyneet alueet muodostavat kuplia, jotka hiljalleen täyttävät koko avaruuden. Jos tämä tapahtuu tarpeeksi äkillisesti ja väkivaltaisesti, niin tapahtumassa saattaa syntyä enemmän hiukkasia kuin antihiukkasia. Kuplien törmätessä voi myös syntyä gravitaatioaaltoja, jotka taivaalle 2030-luvulla nouseva LISA-satelliitti voisi havaita.

Standardimallisssa Higgs kuitenkin jäätyy leppoisasti, niin että ei synny merkittävissä määrin antimaterian ja materian epäsuhtaa eikä gravitaatioaaltoja. Monissa Standardimallin laajennuksissa on toisin, ja asiaa voi ajatella myös niin, että jos LISA näkee tällaisia gravitaatioaaltoja, tämä olisi selvä merkki uudesta fysiikasta. Onkin kiinnostavaa, miten erilaisin tavoin hiukkasfysiikkaa nykyään luodataan: taivaalla kulkevat aika-avaruuden häiriöt voivat kertoa samasta asiasta kuin maanalaisten kiihdytinten törmäyksissä syntyvät hiukkaset.

12 kommenttia “Monta roolia avoinna”

  1. Lentotaidoton sanoo:

    ”Oli erinomaisen perusteltua odottaa, että LHC löytää Higgsin hiukkasen (tai jotain eksoottisempaa), mutta sitä pidemmälle ei ole takeita. Nyt liikutaan avomerellä vailla tietoa siitä, koska maata löytyy, mutta on tietysti lohdullista ajatella, että huomenna horisontissa kajastaa uusi maailma.”

    Taas mielenkiintoinen kirjoitelma. LHC oli varta vasten rakennettu (etenkin kun Tevatron epäonnistui) etsimään ja löytämään Higgsin bosoni. Kun kirjoitit, että nyt ”piireissä” liikutaan mieluusti alhaalta ylös, niin kysymys kuuluu: ei siis ole mitään erityistä teoreettista ”jahdattavaa” – vai voisiko tällainen olla kuitenkin supersymmetria (ja sen kevein hajoamaton hiukkanen)?

    Sitten Higgs ja tyhjö. Kirjoitit aiemmin että tyhjön vakaus liittyy monimutkaisesti Top-kvarkin massan teoriaan. Ymmärrän että tässä yhteydessä ehkä liian ”monimutkainen” selvitettäväksi, mutta voisitko mahdollisesti tulevaisuudessa kirjoittaa aiheesta laajemmin?

    Higgsin kentän jäätyminen tiettyyn arvoon. Perustunee siihen, että kenttä on ilmeisesti inflaation kuluessa viuhtonut ON/OFF useitakin kertoja, mutta löytänyt jossain vaiheessa nykyisen miniminsä. Paitsi raju jäätyminen (246 GeV:ssä) ja mahdollinen aine/antiaine epäsuhta siinä rytäkässä, niin asia liittyy läheisesti yo. tyhjön vakauteen (eli siihen elämmekö väärässä tyhjössä).

    Kaiken kaikkiaan kiintoisa pakkaus tuo Higgs. Ehkä jo HL-LHC tuo asiaan lisävaloa.

    1. Syksy Räsänen sanoo:

      En tarkoita, että ei olisi jahdattavaa, vaan että ei ole mitään vankkaa syytä odottaa, että se on seuraaviem kiihdytinten ulottuvissa.

      LHC:n energioilla Standardimalli ilman Higgsiä ei enää ole matemaattisesti ristiriidaton teoria, eli jotain uutta oli pakko löytyä.

      Supersymmetrian rikkoutumisen energiaa (joka määrää supersymmetristen partnerien massan) voi sen sijaan työntää mielivaltaisen ylös. Odotettiin, että supersymmetria olisi löytynyt jo Tevatronissa, ja sitten LHC:ssä.

      En tiedä palaanko tuohon top-kvarkin massan vaikutukseen. Kaikki fermionit vetävät Higgsin itseisvuorovaikutusta heikommaksi sitä enemmän, mitä korkeammalla energialla ollaan. Top-kvarkki on raskain fermioni, eli vuorovaikuttaa Higgsin kanssa vahvimmin. Se voi vetää Higgsin itseisvuorovaikutusta niin alas, että se muuttuu negatiiviseksi ja potentiaalists tulee epästabiili.

      Higgsin kentän jäätyminen ei tässä liity inflaatioon, vaan tapahtuu paljon myöhemmin, sähköheikon siirtymän aikaan.

  2. Lentotaidoton sanoo:

    ”Higgsin kentän jäätyminen ei tässä liity inflaatioon, vaan tapahtuu paljon myöhemmin, sähköheikon siirtymän aikaan.”

    Juu tämä tietysti näin. Ajatukseni vain oli, että Higgsin kenttä inflaationkin aikana ilmeisesti sahasi on/off. Eli missä rajoissa lienee ollut kentän arvo tuolloin (en ole selvää vastausta löytänyt)? Vaikka sitten jäätyikin nykyiseen arvoonsa sähköheikossa symmetriarikossa.

    1. Syksy Räsänen sanoo:

      Higgsin kentän arvo inflaation aikana riippuu siitä, oliko Higgs vastuussa inflaatiosta (ja jos oli, niin miten tarkalleen) ja inflaation aikaisesta energiatiheydestä. Nollasta eroava kuitenkin.

      [Korjaus: Higgsin kentän arvo voi olla inflaation aikana myös nolla.]

      1. Eusa sanoo:

        Voiko käsityksesi mukaan olla mahdollista Higgsin kentän antipodaali luonne esim. siten, että vastakkaismerkkiset varaukset vuorovaikuttavat sen kanssa vastakkaisin potentiaalein ja siten Higgsin kentän arvo voitaisiin katsoa olevan keskimäärin nolla?

        1. Syksy Räsänen sanoo:

          Tuolla mitä kirjoitit, ei ole mitään tekemistä sen kanssa, miten Higgsin kentän arvo määräytyy. Tämä riittäköön tästä.

  3. Markus sanoo:

    Kvarkeista puheen ollen: R.I.P. Murray Gell-Mann (1929-2019) — yksi 1900-luvun suurimmista yleisneroista ja fyysikoista.

  4. Cargo sanoo:

    ”Enemmän keskitytään rakentamaan tunnetulta pohjalta ylöspäin, ja katsomaan mitä tunnettujen asioiden läheisyydestä voisi löytyä.”

    Havaintojen puutteessa teoreetikot ovat painelleet länget paukkuen syvälle metsään, tai ”deeply into the thicket of hypotheses”, kuten Einstein sanoisi. Eikö yksi lähestymistapa voisi olla kehittää täysin ongelmaton kvanttikenttäteoria, joka selittää nykyisen standardimallin, ja vasta sitten alettaisiin pohtimaan, miten uudet havainnot (mahdollisesti) laajentavat pitävällä matemaattisella perustalla olevaa teoriaa?

    1. Syksy Räsänen sanoo:

      Standardimalli ei ole matemaattisesti ristiriidaton mielivaltaisen korkeilla energioilla. Ei siis ole mitään pitävää matemaattista rakennetta, joka kuvaisi sitä.

      Ks. https://www.ursa.fi/blogi/kosmokseen-kirjoitettua/rajankayntia/

      Kvanttikenttäteorian tarkka matemaattinen muotoilu on kyllä kiinnostava matemaattinen ongelma (yksi Millennium-ongelmista), jonka ratkaisemisesta saattaa olla (tai saattaa olla olematta) hyötyä fysiikallekin.

      Tutkimus etenee aina moneen suuntaan samaan aikaa.

  5. Juha sanoo:

    Kuinka hyviä fyysikot noin yleisesti ovat todennäköisyysteoriassa? Eikö koko alkeishiukkaisfysiikka kannattaisi lähteä rakentamaan todennäköisyyskenttäteorian pohjalta? Tällöin hiukkasen mittaus/interaktio olisi realisaatio satunnaiskentän tn-tiheydestä. Voisiko tällä välttää ongelman, että ”todellisuus on outo”, joka seuraa siitä kun kaikki on rakennettu kaikki klassisen fysiikan pohjalle.

    1. Syksy Räsänen sanoo:

      Kvanttikenttäteoriaa ei ole rakennettu klassisen fysiikan pohjalle. Tämä riittäköön tästä.

Vastaa

Sähköpostiosoitettasi ei julkaista. Pakolliset kentät on merkitty *


Maxwellin tiimalasi

15.5.2019 klo 22.51, kirjoittaja
Kategoriat: Kosmokseen kirjoitettua

Luin Ari Tervashongan gradun Fysiikan referenssiraamin muutos: maxwellilaisen eetterihypoteesin teoriaperinne vuosina 1879–1916. Tämä kiinnostava opinnäytetyö käsittelee nimensä mukaisesti eetteriteorioiden kehitystä sähkömagnetismin kehittäjän James Clerk Maxwellin kuoleman ja yleisen suhteellisuusteorian löytämisen välisenä aikana. Eetteriteorialla ei ole merkitystä nykyfysiikalle mitä sisältöön tulee, mutta sen historia valaisee joitakin piirteitä tieteellisten ideoiden kehityksessä.

Niin fysiikan oppikirjoissa ja kursseilla kuin suurelle yleisölle suunnatuissa kirjoituksissa yleensä esitetään fysiikan kehityksestä yksinkertaistettu versio, missä on karsittu harhapolut ja keskitytään oikeisiin ratkaisuihin. Tämä auttaa ymmärtämään fysiikan teorioiden sisältöä, mutta voi hämärtää käsitystä siitä, miten teorioita kehitetään. Kuten Tervashonka kirjoittaa, tutkimuksen eteneminen ei koostu yksinkertaisista ongelma-ratkaisu-pareista, jotka seuraavat toinen toistaan. Enemmän kuin suoraa punosta, fysiikan kehitys muistuttaa verkkoa, joka kasvaa eri suuntiin, ja jonka osista suurin osa kuihtuu tarpeettomina pois vailla kosketusta todellisuuteen.

Maxwell tunnetaan siitä, että hän löysi sähkön ja magnetismin yhtenäisteorian, sähkömagnetismin. Sen merkitys sekä myöhemmälle fysiikalle että teknologisille sovelluksilla on mittaamaton. Sähkömagnetismi ennusti näkymättömän valon, eli radioaallot, infrapunasäteilyn ja niin edelleen, ja antoi alustan sähköisten ilmiöiden ymmärtämiselle – molemmilla on nykyteknologiassa keskeinen rooli. Se oli myös ponnistuslauta niin suppealle suhteellisuusteorialle, ja siten yleiselle suhteellisuusteorialle, kuin kvanttikenttäteoriallekin – eli molemmille tällä hetkellä perustavanlaatuisille teorioille.

Melko vähän muistetaan sitä, että Maxwell itse hahmotti sähkömagneettiset ilmiöt eetterin kautta. Kun valon ymmärrettiin olevan sähkömagneettista aaltoliikettä, heräsi kysymys minkä aaltoilusta oikein on kyse. (Tarkemmin asiasta täällä.) Jo aiemmin käytössä ollut käsite eetteri valjastettiin tähän rooliin. Eetterin ajateltiin olevan näkymätöntä ainetta, joka täyttää avaruuden kaikkialla ja vuorovaikuttaa tavallisen aineen kanssa, saaden aikaan kaikki sähkömagneettiset ilmiöt ja lämmön.

Maxwellin sähkömagnetismin menestyksen myötä eetterinkin suosio kasvoi. Tervashonka vertaa eetteriteorioiden kehitystä tiimalasiin, jonka uuma on Maxwellin teoria. Ennen Maxwellia oli kaikenlaisia malleja eetteristä, Maxwellin jälkeen suuri osa niistä karsiutui, ja eetteriteorioita rakennettiin lähinnä Maxwellin löytöjen ja ideoiden päälle.

Maxwell oli selittänyt sähkömagnetismia eetterin mekaanisten ominaisuuksien, kuten vieterien ja rattaiden avulla. Tervashongan mukaan Maxwell kuitenkin näki tällaiset ideat vertauskuvallisina tapoina hahmottaa uudenlaista fysiikkaa, eikä halunnut ottaa niitä turhan kirjaimellisesti. Maxwellin sähkömagnetismin teoriassa ei mitään eetteriä tarvitakaan.

Tämä on esimerkki siitä, miten perusteiltaan virheelliset ideat voivat joskus olla hyödyllisiä oikeiden selitysten löytämisessä, kun osaa pitää erillään rakennustelineet ja rakennuksen – ja kun käytössä on uusia havaintoja. Niels Bohrin vuonna 1913 esittämä atomimalli, missä elektronit kiertävät ydintä tiukoilla radoilla on toinen vastaava tapaus, missä yksityiskohdat ovat täysin väärin, mutta ydin ja havaintojen kuvailu oikea.

Maxwellin jälkeen tieteilijät kuitenkin yrittivät ymmärtää eetterin koostumusta sekä selittää havaintoja suoraan eetterimallien avulla, sen sijaan että olisivat vain käyttäneet mekaanisia vertauskuvia apuna niistä riippumattomien teorioiden rakentamiselle. Tämä oli umpikuja, ja ristiriitaisten ideoiden avulla päädyttiin usein selittämään sitä, miksei eetteriä oltu havaittu sen sijaan, että olisi onnistuneesti ennustettu havaintoja ja löydetty uusia ilmiöitä. Toisaalta tämä antoi lukuisia tilaisuuksia kehittää eetterille uudenlaisia ominaisuuksia selitykseksi sille, miksi sitä ei havaittu.

Nykynäkökulmasta eetteritutkimus näyttää hedelmättömältä. Tervashonka kuitenkin muistuttaa, että monet eetteriä tutkineet fyysikot olivat alansa huippua. Hän toteaa, että ideaa ei hylätty vaikeuksienkaan edessä, koska se olisi ollut vielä vaikeampaa kuin jatkaminen. Jos ei ole hyviä ideoita, on pakko käyttää huonoja. Vaihtoehtona olisi myöntää, että ei pääse enää eteenpäin jonkun asian tutkimisessa ja siirtyä tutkimaan jotain muuta. Vaikeutta tuskin vähensi se, että eetterin avulla haluttiin selittää lähes kaikki fysikaaliset ilmiöt. Se, että teorian ennusteet ovat pielessä (tai se ei ennusta mitään) ei riitä sen hylkäämiseen. Kuten yleensä, teoria hylättiin vasta kun keksittiin parempia selityksiä, osittain uusien havaintojen kautta.

Eetteriteorioiden loppuun vaikutti merkittävästi kaksi tapahtumaa. Ensinnäkin Albert Einsteinin esittämä suppea suhteellisuusteoria vuonna 1905 ratkaisi kaikki Maxwellin sähkömagnetismin ja Newtonin mekaniikan väliset ongelmat (muuttamalla jälkimmäistä), joilla oli motivoitu eetterin tutkimista. Toisekseen sähkövirtaa kuljettava elektroni löydettiin, minkä jälkeen sähkön selittämiseen ei tarvittu eetteriä. Kuten Tervashonka toteaa, teorioista luopuminen oli kuitenkin hidasta ja tapahtui eri tahtiin eri paikoissa. Elektronikin liitettiin ensin osaksi eetterimalleja, ja jotkut puolustivat eetteriteoriaa vielä 20 vuotta suppean suhteellisuusteorian löytämisen jälkeen.

Ratkaisevana tekijänä mainitaan usein Michelsonin ja Morleyn vuoden 1887 koe, joka ei löytänyt eetterituulta ja vahvisti epäilyksiä eetterin olemassaolosta. Tervashongan mukaan eetteriä kuitenkin puolustettiin kokeen jälkeen entistä vahvemmin. On vaikea olla vertaamatta tällaista suhtautumista viime vuosikymmenten tutkimukseen säieteoriasta kaikkien vuorovaikutusten yhtenäisteoriana, jota jatketaan teoreettisen edistyksen, ennusteiden ja kokeellisen näytön puutteesta huolimatta. Osa säieteoriayhteisöstä on reagoinut samalla tavalla siihen, että CERNin LHC-kiihdytin ei ole nähnyt merkkejä supersymmetriasta. Kummassakin tapauksessa teoriasta ei ole yhtä kiinnitettyä muotoilua, vaan erilaisia versioita. Jotkut niistä ennustavat, että kokeessa pitäisi näkyä tietynlainen signaali, toisten mukaan mitään ei pitäisi nähdä. Niinpä jos sopivia merkkejä olisi nähty, se olisi tukenut teoriaa, mutta niiden puute ei kumoa teoriaa.

Eetterin historiasta ei voi päätellä mitään siitä, onko säieteoria oikein, mutta se muistuttaa, miten kärkitutkijoiden hartaudella kehittämät kokonaiset viitekehykset voivat olla väärin, ja että ilman uusia havaintoja on vaikea löytää oikeita kysymyksiä, saati vastata niihin.

9 kommenttia “Maxwellin tiimalasi”

  1. Pilkunviilaaja sanoo:

    Eikö pitäisi käyttää hypoteesi-sanaa mielummin kuin teoria? Vai oliko eetteriteorioille olemassa tutkimusnäyttöä?

    1. Syksy Räsänen sanoo:

      Sanaa teoria ei käytetä pelkästään ideoista, jotka on kokeellisesti varmennettu, jos sitä tarkoitat.

      Käytän sanoja malli ja teoria tässä vähän sekaisin, niiden merkityksestä (siten kuin se fysiikassa yleensä ymmärretään) vähän täällä:

      http://www.tiede.fi/artikkeli/blogit/maailmankaikkeutta_etsimassa/kaikki_jarjestyksessa

  2. Eusa sanoo:

    Oireellisinta nykyfysiikassakin on, että parametrien lisäyksille haetaan ”tukea” kuten pimeän aineen metsästys, eikä pitäydytä ennusteiden osoittamisessa vääriksi pätevyysalueen rajoja löytämällä ja siitä sitkeäsi teoriaa kehittämällä.

    Vähintäänkin hämmentävää on ajatusrakennelmien kutsuminen teorioiksi, vaikka ne voivat olla mahdottomia falsifioida eli löytää niitä pätevyysalueen rajoja.

    Kaikki teoriathan ennustavat väärin. Kvanttiteorian huikeat desimaalitarkkuudetkin perustuvat hyvin rajattuun pätevyysalueeseen; vuorovaikutukset kontrolliolosuhteissa.

    Säikeet eivät ole edes malli vaan lähinnä matematiikkaa, eikös vain?

    Ihmisen kaipuu yhteen totuuteen näkyy yhtenäisteorian haikailussa – jos sellainen olisikin, todellisuus perustuisi matematiikalle ja ehkä voisi jokin fraktaalimatematiikka ollakin, mutta tuskin sen keksimisestä juurikaan mitään hyötyä fysiikalle olisi. Nyt esim. kvanttimekaniikka ja yleinen suhteellisuus täydentävät hyvin toisiaan – moni on sitä mieltä, ettei gravitaation kvantittaminen ole edes oikea kysymys.

  3. Mikko Väyrynen sanoo:

    Sana eetteri on teorioissa korvattu sanalla kenttä niin kuin tuossa totesit.
    Vuoden 1887 koe modernisoitiin onnistuneesti kun Gravity Probe B saatiin valmiiksi ja ligo todisti että gravitaatio kulkee samaa nopeutta kuin valo, eli siis samassa kentässä. Ongelmana eetterissä on lähinnä se ettei sitä keksitty 1700 luvulla vaan on esihistoriallista alkuperää ja siihen liitetään myös muita ominaisuuksia, kuten henkiä sekä telepatiaa ja sen myös tiedettiin olevan elossa, tästä lähtökohdasta on vaikeaa lähteä rakentamaan mitään kansan hyväksymää kaupallista tuotteistusta, joten sanaa eetteri ei ole voinut edes mainita joutumatta naurunalaiseksi, propsit siitä

    1. Syksy Räsänen sanoo:

      En sanonut, että sana eetteri on korvattu sanalla kenttä.

  4. Lentotaidoton sanoo:

    Räsänen: ”Eetterin historiasta ei voi päätellä mitään siitä, onko säieteoria oikein, mutta se muistuttaa, miten kärkitutkijoiden hartaudella kehittämät kokonaiset viitekehykset voivat olla väärin, ja että ilman uusia havaintoja on vaikea löytää oikeita kysymyksiä, saati vastata niihin.”

    Useat (ehkä enemmistö) fyysikoistakin silti kirjoittavat (ainakin populaariesityksissä) että tie Standarditeoriasta eteenpäin käy supersymmetrian ja ehkä GUTien kautta tai ilman niitä nimenomaan säieteoriaan/teorioihin. Toinen, mikä aina muistetaan mainita, on Loop Quantum Theory vaikka sen ulottuvuusaste ei edes teoriassa ylety yhtä pitkälle kuin esim M-teoria (eli se ei väitäkään olevansa TOE).

    Onko niin, että yleisölle on väkisinkin kuitenkin annettava JOKIN selitys? Vai johtuuko se siitä, että monilla on näyttöjen puutteesta huolimatta lähes jumalainen usko malleihinsa?

    Standardimallilla on valtava empiirinen todistusaineisto, mutta silti tiedämme sen monet puutteet. Eli tilausta sen ylittäville teorioille toki on. Elämme jännittäviä aikoja, mutta täytyy sanoa, että jännitettävyys on teoriapuolen osalta aikalailla latkuuntunut vuosikymmenien aikana olemattoman näytön puutteessa. Mutta tietenkin sen, että minä en ymmärrä, ei tarvitse olla edes suuntaa antavaa.

    1. Syksy Räsänen sanoo:

      Suurelle yleisölle tarkoitettujen kirjojen kirjoittajat eivät ole edustava otos fyysikoista.

      Säieteoria on epäilemättä alan tutkijoiden keskuudessa suosituin ehdokas kaiken teoriaksi. Yksi syy on se, että parempaakaan vaihtoehtoa ei ole löydetty.

      Mutta en tiedä kuinka moni yhteisön jäsen suhtautuu säieteoriaan tai supersymmetrisiin suuriin yhtenäisteorioihin millaisella varmuudella. Isolla osalla alan tutkimuksesta ei ole kummankaan kanssa mitään tekemistä.

      Jos ei pidä säieteoriaa relevanttina oman tutkimuksen kannalta, niin tuskin lähtee kirjoittamaan kirjaa missä kertoo asiasta.

      Säieteoriaan kriittisesti suhtautuvia kirjojahan on tullut, kuten Lee Smolinin The Trouble with Physics, Peter Woitin Not Even Wrong ja Lawrence Kraussin Hiding in the Mirror.

  5. Erkki Kolehmainen sanoo:

    ”Suurelle yleisölle tarkoitettujen kirjojen kirjoittajat eivät ole edustava otos fyysikoista.”
    Ei tietenkään. Siitä huolimatta, että he ”tuhlaavat” aikaansa suuren yleisön valistamiseeen, he voivat olla merkittäviä myös fysiikan kehityksen kannalta. Esim. Stephen Hawking.

  6. Kari sanoo:

    Tieteen popularisointi on tarpeellinen mutta haastava alue. Iso ongelma siinä on, että tieteen popularisoijat, huonot opettajat ja TV:n tiededokumentit joskus käyttävät alkeellisia mielikuvia ja vertauskuvia jonkin fysiikan ilmiön tai suuruusluokan havainnollistamiseen tavalla, joka voi olla täysin harhaanjohtava tai vielä vaikeammin tajuttava kuin se itse ilmiö. Jospa tässä on merkittävä syy siihen, että maailmalle syntyy sellaisia hassuja liikkeitä kuin ’flat earth society’. Jos tieteellis-kriittisin silmin katsoo niitä selityksiä mitä esim. Maan pallomaisuuden puolesta annetaan, niin usein ne eivät ole kovin hyviä, jos niitä tiedekriittisin silmin lukee. Lisäksi niihin liittyy tarpeetonta ”vastapuolen” halventamista. Kukapa meistäkään haluaisi kuunnella sellaista opettajaa, joka luennon alussa haukkuu kuulijansa mahdollisimman loukkaavin tavoin?

    Mitä tieteen teorioiden hylkäämiseen tulee, tieteen historia on mielestäni näyttänyt, että virheellinen teoria ei kuole sen takia, että joku todistaa sen vääräksi vaan sen takia, että sukupolvi joka siihen uskoi, kuolee pois.

Vastaa

Sähköpostiosoitettasi ei julkaista. Pakolliset kentät on merkitty *

Toismaailmallinen arki

29.4.2019 klo 13.20, kirjoittaja
Kategoriat: Kosmokseen kirjoitettua , Kosmologia

Gravitaatioaaltoja kuuntelevien laitteiden LIGO ja Virgo kolmas havaintokausi alkoi huhtikuun 1. päivä. Toinen kausi päättyi vuoden 2017 elokuussa, ja laitteita päivitettiin puolitoista vuotta herkkyyden kasvattamiseksi. Gravitaatioaallot ovat sitä heikompia mitä kauempana niiden lähde on. Toisin sanoen mitä tarkempi laite on, sitä kauempaa se naaraa maailmankaikkeutta.

Kahdeksan kuukautta kestäneellä toisella havaintokaudella LIGO/Virgo näkivät kahdeksan gravitaatioaaltoa, noin yhden kuukaudessa. Nyt kuukaudessa on saatu haaviin jo viisi mahdollista havaintoa, noin yksi viikossa. Kolmannen kauden on määrä kestää ainakin vuoden, joten luvassa on kymmeniä havaintoja mustien aukkojen törmäyksistä, varmaan myös neutronitähtien törmäyksistä, ja kenties nähdään uudenlaisiakin signaaleja.

Varaus ”mahdollinen” yllä liittyy siihen, että LIGO/Virgo on alkanut ilmoittaa havainnoista nopeasti, ennen kuin niiden tulkintaa on täysin varmistettu. Tarkoituksena on, että muut koeryhmät voivat suunnata teleskooppinsa oikeaan kohtaan taivasta mahdollisimman pian etsiessään gravitaatioaallot synnyttäneessä törmäyksessä ja sen jälkimainingeissa syntynyttä valoa ja muuta sähkömagneettista säteilyä. Toistaiseksi sellaista on nähty vain kerran, lokakuussa 2017, kun havaittiin (luultavasti) kahden neutronitähden törmäys räväkästi gravitaatioaaltojen lisäksi gammasäteillä, näkyvällä valolla, röntgensäteillä ja radioaalloilla. Mustien aukkojen törmäyksistä ei odoteta syntyvän havaittavissa määrin sähkömagneettista säteilyä.

LIGOn/Virgon ilmoitukset mahdollisista gravitaatioaalloista ovat julkisia. On jopa tehty ilmainen sovellus, jolla voi saada niistä puhelimeensa viestin yhtä aikaa tiedeyhteisön kanssa.

Olen kirjoittanut siitä, miten gravitaatioaaltojen kasvava määrä mahdollistaa uudenlaista tietoa antavan tilastollisen tarkastelun ja auttaa muun muassa selvittämään, ovatko mustat aukot syntyneet tähtien romahduksessa vai onko niiden taustalla sen sijaan kosminen inflaatio tai muu muinaisen maailmankaikkeuden ilmiö. Mutta mustien aukkojen gravitaatioaaltojen arkistuminen tuntuu merkitykselliseltä muutenkin kuin tulevia tieteellisiä tuloksia ajatellen.

Muutama vuosikymmen sitten ajatus siitä, että voi saada mukana kantamaansa pieneen levyyn missä tahansa päin Maata, tai vaikka kilometrien korkeudessa ilmassa, lähes reaaliaikaisia tiedotteita siitä, että Maapallon läpi pyyhkäisee miljardisosan miljardisosan tuhannesosan suuruisia värähtelyitä, jotka ovat peräisin kymmenien, satojen tai tuhansien miljoonien valovuosien päässä menneisyydessä tapahtuneista mustien aukkojen törmäyksistä olisi vaikuttanut epärealistisen scifistiseltä jopa vuoden 2020 tienoilla.

Sata vuotta sitten, kun yleisen suhteellisuusteorian ensimmäistä ennustetta valon taipumisesta testattiin, välineet mustien aukkojen tieteen ymmärtämiseen olivat uusia, mutta niiden havaitsemiseen ja siitä kertomiseen käytetty teknologia olisi tuntunut niin uskomattomalta, että siitä hädin tuskin uskaltaisi unelmoida.

Muutama vuosisata sitten koko asia mustista aukoista, gravitaatioaalloista sekä näkymättömään valoon ja sähköön pohjaavista kämmenen kokoisista viestimistä, jotka on koottu metallista, lasista ja muovista, ja jotka viiveettä välittävät tietoa ympäri maailmaa olisi ollut käsityspiirin ulkopuolella. Kukaan ei tuolloin pystynyt kuvittelemaan nykyelämämme arkisia teknologisia piirteitä sähköstä ja tietokoneista lähtien, saati tieteen paljastamaa epäinhimillistä totuutta maailmasta. Kun ei ollut viitekehystä tällaisten asioiden ymmärtämiseen, edes niiden reunoja ei olisi voinut hahmottaa muuten kuin epäjärjestelmällisen taikuuden tai irrallisten ihmeiden kautta.

Koska asioita hahmotetaan suhteessa toisiinsa ja tiede sekä teknologia kasvavat kanssamme samaa tahtia, taianomaisten tapahtumien tulo todeksi ei ole saanut elämäämme tuntumaan toismaailmalliselta, vaan se on muuttanut ihmeellisen arkiseksi. Ihmetyksen tunne vaatii pysähtymistä ja vertailukohdan hakemista jokapäiväisen ulkopuolelta, menneisyyden ottamista kiintopisteeksi kuin tähtitaivaan, jonka etäinen vieraus asettaa maanpäällisen elämän eri mittoihin.

Yksi kommentti “Toismaailmallinen arki”

  1. Historiasta puheen ollen, tässä on vuoden 1978 Arkhimedes-lehden Kaarle Kurki-Suonion kirjoittama kirja-arvostelun Raimo Keskisen ja Heikki Ojan kirjasta ”Mustaa aukkoa etsimässä”: https://www.mv.helsinki.fi/home/kurkisuo/6.3.C/78-Arkhimedes-MustatAukot.pdf . Kyseisen kirjan takakannessa oli runo jossa oli mm.:

    -Weberin käy vehkeet soimaan, kun tähden nauttii aamupalaksi ja lounaaksi on pieni galaksi…

    Weberin vehkeet eivät ihan toimineet, mutta nyt vähän hienommat vehkeet soivat ja ”aika-avaruuden seismologia” on alkanut, jolla kuullaan kuinka ne maailmankaikkeuden kovat luut kolisevat eikä nähdä vain lihoja kuten sähkömagneettisilla aalloilla.

Vastaa

Sähköpostiosoitettasi ei julkaista. Pakolliset kentät on merkitty *


Marssi ja päivät

18.4.2019 klo 15.48, kirjoittaja
Kategoriat: Kosmokseen kirjoitettua

Puhun lauantaina 4.5. joskus kello 13-15 välillä tiedemarssilla Tiedekulmassa omasta tutkimuksestani, sen merkityksestä ja tieteen merkityksestä yhteiskunnalle yleensä. Sanonpa muutaman sanasen tiedemarssinkin merkityksestä. Kokoontuminen tiedemarssille alkaa kello 11.30.

Puhun lauantaina 24.8. kello 13.00-13.45 Kuopion tieteen päivillä osana Kuopio juhlii -tapahtumaa otsikolla ”Rohkeus ja rakenteet – miksi on turhaa etsiä seuraavaa Einsteinia”. Luvassa yleisen suhteellisuusteorian ja kosmologian historiaa ja huomioita tieteen edistämisestä. Tiivistelmä täällä (Kuopion esitys tosin on laveampi, ja ehkä eri tavalla painotettu.). Minua haastateltiin aiheen tiimoilta Helsingin tieteen päivien yhteydessä.

3 kommenttia “Marssi ja päivät”

  1. Uutinen T&A sivustolla koskien LIGO:a: ”Uudistusten ansiosta LIGOn suorituskyvyn arveltiin nousevan niin paljon, että se pystyisi huomaamaan massiivisten kappaleiden törmäyksiä jopa viikoittain. Nyt julkaistut tulokset ovat siis selvä osoitus siitä, että LIGOn päivitys on onnistunut. Tie on auki suureen määrään havaintoja.”

    Tuo uutinen, jos pitää paikkansa, kuulostaa erittäin hienolta. Olen tähän mennessä yllättynyt positiivisesti siitä kuinka paljon johtopäätöksiä jo ensimmäisistä muutamasta havainnosta saatiin, vaikkakaan se ei ehkä ollut yllätys alan tutkijoille.

  2. Cargo sanoo:

    Eikö puheissa voisi välillä painottaa mielikuvituksen sekä intuition merkitystä ja todeta, että uudelle Einsteinille on aina tilausta? Siis ilon eikä kyynisyyden kautta 🙂

    Einstein oli alansa tunteva tiedemies, joka sovelsi intuitiotaan havaittuihin ilmiöihin ja sattui löytämään oikeita selityksiä. Välillä taas Einsteinin intuitio meni myös puihin, esim. EPR tai klassinen kaikenteoria. Näin ajateltuna mahdollisia einsteinejä on maailma täynnä, sillä kenenkään ei tarvitse maalata Einsteinistä kuvaa tiedemaailmasta eristäytyneenä supernerona.

    Paraatiesimerkki Einsteinin intuitiosta oli valon hiukkasluonteen ”löytäminen” termodynamiikan avulla: Einstein havaitsi, että mustankappaleen säteilyjakauman entropia muuttuu tilavuuden suhteen samalla tavalla kuin atomeista koostuvalla kaasulla. Yhdistämällä tämä Planckin kvanttihypoteesiin saattoi tehdä hämmentävän oletuksen, että säteily koostuukin hiukkasista. Einstein sovelsi ideaansa valosähköiseen ilmiöön ja pokkasi ansaitusti Nobelin. Muut Einsteinin ideat saattoivat syntyä tiiviimmässä vuoropuhelussa ympäröivän tiedemaailman kanssa.

    1. Syksy Räsänen sanoo:

      Ilolla ja kyynisyydellä ei ole mitään tekemistä asian kanssa.

Vastaa

Sähköpostiosoitettasi ei julkaista. Pakolliset kentät on merkitty *

Äärimmäisyyden reunalta

12.4.2019 klo 21.49, kirjoittaja
Kategoriat: Kosmokseen kirjoitettua , Kosmologia

Keskiviikkona Event Horizon Telescope (EHT) -tutkimusryhmän ottama kuva galaksin M87 mustan aukon tapahtumahorisontin liepeiltä levisi kulovalkean tavoin ympäri maailmaa. Tämä oli historiallinen hetki, ensimmäinen kerta kun ihmiskunta on nähnyt valon läheltä yleisen suhteellisuusteorian ennustamaa viimeistä rajaa. Valokuva tuotiin julki sata vuotta sen jälkeen kun ensimmäinen havainto valon taipumisesta varmisti yleisen suhteellisuusteorian. Mustien aukkojen salaperäinen maine ja säväyttävä kuva aiheuttivat samanlaisen innostuksen kuin Higgsin hiukkasen löytäminen vuonna 2012 ja gravitaatioaaltojen suora havaitseminen vuonna 2016.

Kurkistan tässä kuvan taakse ja hahmottelen mitä se meille kertoo. EHT:n tiivistelmäartikkeli löytyy täältä, ja EHT:n suomalaisen jäsenen Tuomas Savolaisen kokemuksesta voi lukea täällä.

EHT-ryhmä teki havainnot ympäri Maapalloa sijaitsevilla teleskoopeilla viime vuoden 2017 huhtikuussa. Kirjoitin aiheesta viime toukokuussa, jolloin arveltiin, että kuva olisi valmis viime vuoden loppuun mennessä. EHT on teleskooppien verkosto, ja jokainen sen teleskooppien pari näkee pienen siivun taivasta. Näiden kuvien puhdistaminen, analysoiminen ja yhdistäminen sekä väliin jäävien alueiden täyttäminen todennäköisimmällä kuvanjatkeella kesti hieman odotettua kauemmin.

Galaksi M87 on 55 miljoonan valovuoden päässä, eli kosmologisesti ajateltuna samassa korttelissa. M87 on lähitienoon massiivisin galaksi, ja sen keskellä mollottava kuuden miljardin Auringon painoinen musta aukko on niin ikään naapuruston isoin. Keskustasta pursuaa 200 000 valovuotta pitkä hiukkassuihku, joka havaittiin ensimmäisen kerran vuonna 1918. Tällaiset valtavat suihkut syntyvät kun aine syöksyy mustaan aukkoon kierteessä kuin vesi kylpyammeen aukkoon. Aineen pyöriessä kehittyy vahvoja magneettikenttiä, jotka kiihdyttävät hiukkasia korkeisiin energioihin. Varhaisessa maailmankaikkeudessa mustiin aukkoihin virtaa ainetta vuolaana koskena. Tämä synnyttää erittäin voimakkaita suihkuja, ja tällaiset mustat aukot tunnetaan nimellä kvasaari. Kvasaareja on mitattu satojatuhansia, mutta ne ovat liian kaukana, jotta kiekoista saisi selvää.

Galaksin M87 hiukkassuihku on hillitympi, mutta sen perusteella odotettiin, että mustaa aukkoa kiertävästä ainekiekosta saataisiin viimein kuva. Tässä ei jouduttu pettymään. Kuvan valodonitsi syntyy elektroneista, jotka kiertävät mustan aukon nielua lähes valonnopeudella miljardin asteen lämpötilassa ennen suistumistaan nieluun. Donitsin paksuus kuvassa johtuu teleskooppien erotuskyvyn rajoituksista: kiekko on huomattavasti ohuempi kuin miltä näyttää. Alapuoli on kirkkaampi, koska siellä elektronit liikkuvat meitä kohti.

Kuvassa näkyvä tumma keskus on nimetty ”mustan aukon varjoksi”. Koska musta aukko ei käytännössä lähetä eikä heijasta säteilyä (Hawkingin säteily on aivan liian heikkoa havaittavaksi), se näyttäytyy tummana alueena kirkkauden keskellä. Toisin kuin miltä kuvasta saattaa näyttää, keskusta ei itse asiassa ole täysin musta, sen kirkkaus on noin 10% kiekon kirkkaudesta. Tämä on odotettavissa ensinnäkin siksi, että musta aukko taivuttaa valoa niin että sen taakse voi nähdä. Toisekseen, mikä tärkeämpää, EHT:n erotuskyky ei aivan riitä keskustan erottamiseen ympäristöstä, joten kiekon kirkkaus tuhraantuu vähän sisemmällekin.

Galaksin M87 keskustan musta aukko on noin Aurinkokunnan kokoinen, kuten sarjakuva xkcd havainnollistaa. 55 miljoonan valovuoden etäisyydeltä se näyttää kuitenkin pieneltä kuin soranjyvä Atlantin takana. EHT:n erotuskyky on vähän keskustan mustaa aluetta huonompi. Mustan aukon ympärillä oleva alue, josta valo päätyy aukkoon, on puolestaan noin 2.6 kertaa tapahtumahorisontin kokoinen. On kutkuttavaa päästä näkemään näinkin lähelle tapahtumahorisonttia, ja EHT on tekninen taidonnäyte. Tuhansien kilometrien päässä olevien teleskooppien etäisyys pitää tuntea millimetrin tarkkuudella, ja ne keräsivät dataa samaa tahtia kuin CERNin LHC-kiihdyttimen kaikki kokeet yhteensä, 32 PB (eli 32 miljoonaa gigabittiä) sekunnissa.

Tieteellisesti tästä ensimmäisestä havainnosta ei kuitenkaan opittu paljon. EHT-ryhmä teki 43 kappaletta simulaatioita erilaisista kertymäkiekoista mustan aukon ympärillä, joissa huomioitiin yleisen suhteellisuusteorian kaikki hienoudet, mutta EHT:n erotuskyvyllä ne näyttävät melkein kaikki jokseenkin samalta. Asian voi ilmaista myös niin, että mallit ovat hyvin ennustusvoimaisia: jos havainnot olisivat näyttäneet jotain muuta, sitä olisi ollut vaikea selittää säätämällä kiekon yksityiskohtia. Mutta nyt ei nähty mitään odotuksista poikkeavaa.

Mustien aukkojen tapahtumahorisontin läheisyydestä tehdyillä havainnoilla voi testata yleisen suhteellisuusteorian ennusteita. Tämän ensimmäisen havainnon antamat rajat ovat kuitenkin hyvin heikkoja. Toistaiseksi voidaan sanoa vain, että mustan aukon pyörimisestä aiheutuva gravitaatiokentän poikkeama pallosymmetriasta voi olla korkeintaan neljä kertaa niin iso kuin mitä suhteellisuusteoria ennustaa. Näillä havainnoilla ei voida sulkea pois moniakaan vaihtoehtoja mustille aukoille.

Vaikka EHT-ryhmä summaa muuttaneensa tapahtumahorisontin ”matemaattisesta käsitteestä fyysiseksi olioksi jota voi tutkia toistuvilla tähtitieteellisillä havainnoilla”, mustien aukkojen törmäyksistä syntyneiden gravitaatioaaltojen havaitseminen on jo antanut tarkempia rajoja mustien aukkojen ominaisuuksille. Ryhmä on kuitenkin oikeassa siinä, että se on avannut mahdollisuuden tutkia samoja mustia aukkoja useampia kertoja yhä tarkemmin, siinä missä gravitaatioaallot kiitävät ohi valonnopeudella.

Lähitulevaisuudessa sopii odottaa tietoa kiekon valon polarisaatiosta, nyt julkaistiin vasta data valon kirkkaudesta. Polarisaatio kertoo kiekon magneettikentästä. Lisäksi on luvassa kuva EHT:n toisesta kohteesta, Linnunradan keskustan mustasta aukosta. Se on vaikeampi  nähdä, koska Linnunradan keskustassa ja matkalle sinne on kaikenlaista roskaa. Lisäksi Linnunradan mustan aukon ympäristö muuttuu nopeammin kuin galaksin M87 mustan aukon ja on siksi vaikeammin kuvattava.

Kuten ensimmäinen suora havainto gravitaatioaalloista, ensimmäinen valokuva mustasta aukosta on enemmän lupaus tulevasta kuin itsessään mullistava löytö.

Päivitys (16/04/19): Havaintojen vuosi korjattu.

9 kommenttia “Äärimmäisyyden reunalta”

  1. Adam sanoo:

    Onko tämä kuva nyt sitten (täysin) kiistaton todiste mustan aukon olemassaolosta?

    1. Syksy Räsänen sanoo:

      Ei. Kuten tekstissä kirjoitan: ”Näillä havainnoilla ei voida sulkea pois moniakaan vaihtoehtoja mustille aukoille.”

      Virkkeessä linkatussa merkinnässä on yksi esimerkki vaihtoehdoista: https://www.tiede.fi/blogit/maailmankaikkeutta_etsimassa/rajapintojen_kosketuksia

  2. Mika sanoo:

    Onko maapallolta nähtävissä yhtäkään mustaksi aukoksi arveltua kohdetta, josta olisi edes teoriassa mahdollista saada vastaavaa kuvaa näkyvän valon aallonpituuksilla? Ilmeisesti ei, tai muuten sitä olisi varmaan jo yritetty?

    1. Syksy Räsänen sanoo:

      En ole varma, ei käsittääkseni nykyisellä teknologialla.

      En tunne teleskooppeja juurikaan, näkyvillä aallonpituuksilla on enemmän ongelmia ilmakehän kanssa, mutta interferometriaa (eli eri teleskooppien kuvien yhdistämistä) tehdään kyllä niilläkin.

      M87:n keskustasta en ole varma, mutta Linnunradassa asiaan vaikuttaa myös se, että galaksimme on läpinäkyvämpi radioaalloille kuin nökyvälle valolle, etenkin keskustan tienoilla.

    2. EHT:n aallonpituus oli 1.3mm ja efektiivinen halkaisija sama kuin Maan. Optisella aallonpituudella 500nm vastaavan kulmaresoluution omaavan teleskoopin halkaisijan pitäisi olla niinkin iso kuin 5 km. Ei taida olla ihan lähitulevaisuuden juttu, tosin avaruusteleskooppien muodostelmalennolla se saattaisi onnistua, ja tarkkaa muodostelmalentoa kehitetään Lisa-gravitaatioaaltoteleskooppia varten. Avaruusinterferometrejä (Darwin, TPF) suunniteltiin 10-15 vuotta sitten, mutta jossain vaiheessa ne karsittiin pois.

  3. Jari Toivanen sanoo:

    Mieleen tulee vanha sanonta: ”suomalainen ei usko ennenkuin näkee”. Mustaan aukkoonkaan ei ilmeisesti voi uskoa, ennenkuin saadaan todisteet näkyvän valon aallonpituusalueella.

    1. Syksy Räsänen sanoo:

      Aallonpituudella ei ole tässä asiassa merkitystä, vaan sillä miten mustista aukoista poikkeavien mallien ennusteita pystytään rajoittamaan.

  4. Janne Savela sanoo:

    Eikös EHT:n havainnot tehty 2017 huhtikuussa? Tuo tekstin alussa olevan EHT:n tiivistelmäartikkelin linkkikään ei näyttäisi toimivan.

    Voitko Syksy vielä vastata muutamaan kysymykseen singulariteeteistä, vaikkeivat ne varsinaisesti tämän tekstin aiheena olleetkaan?

    Onko alkuräjähdyksen- ja mustan aukon singulariteetti samankaltainen? Tiedätkö ovatko tutkijat pohtineet voivatko nämä olla jotenkin yhteydessä toisiinsa esim. siten, että mustan aukon singulariteetti olisi alku uudelle universumille? Huomioiden myös, että mustia aukkoja on eri kokoisia (tähdenmassaiset ja galaksiyhtimissä olevat), voisiko meidän universumimme olla vielä suurempi musta aukko, jonka sisällä olemme?

    1. Syksy Räsänen sanoo:

      Tosiaan, mitähän olenkin löpertänyt. Kiitos, korjasin.

      Mustan aukon ja maailmankaikkeuden alun singulariteetit ovat erilaisia, ja nykynäkökulmasta singulariteetit ovat osoituksia yleisen suhteellisuusteorian pätevyysalueen ylittymisestä. Ideoita siitä, että olisimme mustan aukon sisällä on toki esitetty, mutta ne ovat hyvin spekulatiivisia.

Vastaa

Sähköpostiosoitettasi ei julkaista. Pakolliset kentät on merkitty *


Yhtenäisyyden kuorimista

7.4.2019 klo 18.50, kirjoittaja
Kategoriat: Kosmokseen kirjoitettua , Kosmologia

Kirjoitin Helsingin opettajien ammattiyhdistyksen lehteen Rihveli 1/2019 artikkelin kvanttigravitaatiosta otsikolla Yhtenäisyyden kuorimista. Sen viimeinen kappale on tämä:

Inflaation kautta olemme jo saaneet käsityksen siitä, mitä maailmankaikkeuden ensimmäisen sekunnin perukoilla tapahtui ja hahmottaneet, että kaikki näkemämme –tähdet, planeetat, DNA, ihmiset, koko inhimillinen kulttuuri– on lähtöisin sattumanvaraisista kvanttivärähtelyistä. Kokonaisen kvanttigravitaatioteorian, saati kaiken teorian, löytäminen luultavasti myllertäisi käsityksemme kosmisesta historiasta vielä perinpohjaisemmin, ja asettaisi ajan, avaruuden ja aineen aivan uuteen valoon. Toistaiseksi vuoren huippu on sumun peitossa, eikä tiedetä mikä polku sinne vie.

3 kommenttia “Yhtenäisyyden kuorimista”

  1. 7v sanoo:

    onko olemassa minkäänlaista
    arviota siitä milloin tuo puuttuva kvanttigravitaatioteoria
    voisi löytyä?

    Fermilabin videolla don lincoln

    https://www.youtube.com/watch?v=9LGBo7dLgYk

    kertoo että kaiken teoriaa tutkiessa nykytekniikalla jo
    hiukkaskiihdyttimen renkaan halkasija tulisi
    olla 1000 valovuotta..

    taitaa mennä tuhansia vuosia että joku ryhmä onnistuu?

    1. Syksy Räsänen sanoo:

      Mahdotonta sanoa. Mutta on olemassa tapoja luodata korkeita energioita ilman hiukkaskiihdyttimiä (esimerkiksi kosmisen mikroaaltotausta avulla), niin että tuo kommentti kiihdyttimen hurjasta säteestä ei ole kovin oleellinen.

      Lisää aiheen tiimoilta täällä:

      https://www.ursa.fi/blogi/kosmokseen-kirjoitettua/aika-avaruuden-atomit/

      https://www.ursa.fi/blogi/kosmokseen-kirjoitettua/kaikki-tai-ei-mitaan/

      https://www.ursa.fi/blogi/kosmokseen-kirjoitettua/miksi-kaiken-teorialla-on-merkitysta/

      http://www.tiede.fi/blogit/maailmankaikkeutta_etsimassa/maljan_jaljilla

  2. 7v sanoo:

    ok. tietämättömänä luulin että vain huomisen
    hiukkaskiihdyttimillä voidaan
    tuollanen asia todentaa.
    pitää nuo linkit lukea.

Vastaa

Sähköpostiosoitettasi ei julkaista. Pakolliset kentät on merkitty *


Lähellä ja kaukana

28.3.2019 klo 20.21, kirjoittaja
Kategoriat: Kosmokseen kirjoitettua , Kosmologia

Maailmankaikkeuden laajenemisnopeus on eräs kosmologian keskeisiä suureita. Vuonna 1927 Georges Lemaître osoitti, että yleisen suhteellisuusteorian ennustaman maailmankaikkeuden laajenemisen takia galaksit etääntyvät toisistaan nopeudella, joka on verrannollinen niiden etäisyyteen. Samalla Lemaître määritti havainnoista nopeuden ja etäisyyden välisen verrannollisuuskertoimen. Tämä oli yksi modernin kosmologian ensimmäisiä askelia.

Lemaîtren löytämä laajenemislaki tunnetaan nykyään Hubblen lakina ja siinä oleva vakio Hubblen vakiona, muistaen Edwin Hubblea. Kaksi vuotta myöhemmin Hubble nimittäin löysi havainnoista saman lain ja määritti saman kertoimen. Hubblea on sittemmin tituleerattu ”laajenevan maailmankaikkeuden isäksi”, mikä on sikäli eriskummallista, että hän kuolemaansa asti kiisti sen, että havainnot osoittivat maailmankaikkeuden laajenevan.

Lemaître määritti Hubblen vakion arvoksi arvoksi 625 km/s/Mpc, Hubble 500 km/s/Mpc, missä Mpc on noin 3 miljoonaa valovuotta. Tämä tarkoittaa sitä, että jos kahden galaksin etäisyys on kolme miljoonaa valovuotta, niiden väliin tulee joka sekunti 500 km lisää etäisyyttä; jos etäisyys on kuusi miljoonaa valovuotta, etäisyys kasvaa 1 000 km sekunnissa, ja niin edelleen.

Periaatteessa Hubblen vakion arvon selvittäminen on helppoa: katsotaan, millä nopeudella galaksi etääntyy meistä ja jaetaan sen etäisyydellä. Nopeus on helppo määrittää valon punasiirtymästä, mutta etäisyyden mittaaminen on vaikeampaa. Vielä 1900-luvun alussa luultiin, että paljain silminkin näkyvä Andromeda ja muut galaksit ovat ”tähtisumuja” Linnunradassa. Vuonna 1924 Hubblen oli osoittanut, että ne ovat liian kaukana ollakseen osa Linnunrataa. Havainto osoitti, että Linnunrata on vain yksi galaksi muiden joukossa, mullisti käsityksen maailmankaikkeudesta ja valmisti tien laajenemisen löytämiselle.

Mistä tietää onko Andromeda pieni ja lähellä vai iso ja kaukana? Yksi keino on verrata samanlaisia kohteita täällä kotogalaksissa ja Andromedassa. Taivaan tähdet näyttävät sitä himmeämmiltä, mitä kauempana ne ovat. Vertaamalla sitä, miten himmeältä läheinen ja kaukainen tähti näyttävät voi määrittää niiden etäisyyksien suhteen. Jos tietää läheisen tähden etäisyyden, tästä saa selville kuinka kaukana toinen tähti on.

Menetelmän heikkous on se, että tähtien pitää olla tarpeeksi samanlaisia. Lemaître ja Hubble vertasivat virheellisesti erilaisia tähtiä. Heidän käyttämänsä muiden galaksien tähdet olivat luonnostaan Linnunradan vertailutähtiä kirkkaampia, joten he aliarvioivat galaksien etäisyyden. Senkin jälkeen kun virhe korjattiin, on Hubblen vakion historia ollut täynnä epävarmuutta ja kiistoja. Pitkään Hubblen vakiosta oli kaksi kilpailevaa arviota, 50 km/s/Mpc ja 100 km/s/Mpc.

1990-luvulla tilanne muuttui, kun tehtiin entistä monimuotoisempia ja tarkempia kosmologisia havaintoja, ja kohteiden ominaisuudet ymmärrettiin tarkemmin. Yksi tärkeimpiä uusia havaintokohteita oli kosminen mikroaaltotausta, jonka epätasaisuudet COBE-satelliitti mittasi vuonna 1992. (Tämä palkittiin Nobelilla vuonna 2006.)

Kosminen mikroaaltotausta on näkymätöntä valoa, joka lähti matkaan aineen ja valon erotessa toisistaan maailmankaikkeuden ollessa 380 000 vuotta vanha, 14 miljardia vuotta sitten. Kuten auringonpilkut näyttävät sitä pienemmiltä, mitä kauempana Aurinko on, mikroaaltotaustan täplät näyttävät sitä pienemmiltä, mitä pidemmän matkan se on kulkenut. Kosmisen mikroaaltotaustan täplien koko on siis kääntäen verrannollinen sen kulkemaan matkaan, mikä taasen riippuu siitä, miten maailmankaikkeus on laajentunut, eli Hubblen vakiosta.

Menetelmän heikkous on se, että laajenemisnopeus muuttuu ajan myötä – aiemmin maailmankaikkeuden laajeneminen hidastui, viimeisen muutaman miljardin vuoden aikana se on kiihtynyt. Etäisyyden ja Hubblen vakion määrittämiseksi kosmisesta mikroaaltotaustasta pitää tietää, miten laajenemisnopeus on muuttunut. Yleisessä suhteellisuusteoriassa maailmankaikkeuden ainesisältö määrää sen laajenemisen. Pitää siis tietää millaista ainetta maailmankaikkeudessa on ja kuinka paljon. Suurin tähän liittyvä epävarmuus on se, millaista kiihtyvän laajenemisen aiheuttava pimeä energia on – vai onko kiihtymisellä joku muu selitys.

Asian voi nähdä myös parhain päin: koska Hubblen vakion arvo riippuu pimeän energian mallista, niin sen mittaamalla saa tietoa pimeästä energiasta. Fyysikoita ei yleensä kiinnostakaan Hubblen vakion, Higgsin massan tai muiden suureiden arvo sinänsä, vaan se mitä niistä voi päätellä siitä, millainen maailma on.

Kun olettaa että kiihtyvästä laajenemisesta on vastuussa tyhjön energia, mikä on yksinkertaisin selitys, niin Planck-satelliitin mikroaaltotaustan mittausten mukaan Hubblen vakio on 67.4 km/s/Mpc, tarkkuudella 0.5 km/s/Mpc.

Hubblen vakiota mitataan yhä myös samaan tapaan kuin Lemaîtren ja Hubblen aikaan, vertaamalla lähellä ja kaukana näkyviä tähtiä. Nykyään käytetään useampia askelmia: lähitienoiden tähtien avulla selvitetään naapurigalaksien etäisyydet, ja niissä räjähtävien supernovien avulla määritetään kaukaisempien galaksien etäisyydet. Viime viikolla ilmestyi uusin tutkimus näin päätellystä Hubblen vakion arvosta: 74.0 km/s/Mpc, tarkkuudella 1.4 km/s/Mpc. Tutkimusta johti Adam Riess, joka on etäisyyksien ja supernovien tutkimisen veteraani, ja sai vuonna 2011 kolmanneksen Nobelin palkinnosta kiihtyvän laajenemisen löytämisestä supernovien avulla.

Kosmisen mikroaaltotaustan ja läheisten galaksien avulla määriteltyjen Hubblen vakion arvojen ero on paljon virherajoja isompi. Todennäköisyys sille, että kyseessä on tilastollinen sattuma on yksi miljardista. Riessin ja kumpp. tutkimus on viimeisin sana eri ryhmien tutkimuksissa, jotka ovat syynänneet lähialueiden etäisyysmittauksia. Mitä tarkemmin asiaa on katsottu, sitä isommaksi ero Planckin tulokseen on kasvanut.

Riess ja kumpp. ehdottavat, että ratkaisuna olisi muutokset kosmisen mikroaaltotaustan muodostumisen aikoihin. Jos maailmankaikkeus laajenee 380 000 vuoden ikäisenä eri tavalla, mikroaaltotaustaan voi syntyä odotettua pienempiä täpliä, jolloin etäisyys näyttää isommalta kuin mitä onkaan.

Samalla pitäisi kuitenkin selittää muut mittaukset Hubblen vakiosta. Maailmankaikkeuden laajenemista on esimerkiksi selvitetty galaksien iän avulla. Idea on yksinkertainen. Kun mitataan galaksista tulevaa valoa, sen punasiirtymä kertoo, paljonko maailmankaikkeus on laajentunut sen jälkeen kun valo lähti galaksista. Kun määritetään galaksien ikä ja toistetaan mittaukset eri galakseille, saadaan selville maailmankaikkeuden koon muutos ajan myötä, ja siitä laajenemisnopeus. Näin saatu arvo Hubblen vakiolle on riippumaton kosmisesta mikroaaltotaustasta. Se kuitenkin sopii yhteen Planckin kanssa, ei läheisten supernovien. Tässäkin mittauksessa voi tosin olla omat ongelmansa.

Toinen mahdollisuus on se, että maailmankaikkeuden kiihtyvästä laajenemisesta ei olekaan vastuussa tyhjön energia, joten maailmankaikkeus laajenee eri tavalla kuin mitä odotetaan. Tämä voisi sopia yhteen kaikkien havaintojen kanssa. Mitään vakuuttavaa ideaa tästä ei kuitenkaan ole toistaiseksi ole esitetty, ja muitakin vaihtoehtoja on tutkittu.

Kosmologit ovat lämmenneet Planckin ja lähitienoon mittausten ristiriidan merkitykselle hitaasti, kenties Hubblen vakion poukkoilevan historian takia. Muutamassa vuodessa se on kuitenkin tasaisesti noussut kosmologian suurimmaksi ongelmaksi mitä havaintoihin tulee, ja nyt on ainakin selvää, että kyseessä ei ole sattuma. Jos vastuussa on etäisyyden määrittämiseen liittyvä virhe, se ei ole aivan yksinkertainen, kun tarkka seulonta ei ole sitä vielä löytänyt. Jos tämä sen sijaan on merkki uudenlaisesta fysiikasta, se olisi merkittävin löytö kosmologiassa ainakin kahteen vuosikymmeneen.

36 kommenttia “Lähellä ja kaukana”

  1. Tommi Tenkanen sanoo:

    Tämäpä sattui hauskasti: istun juuri luentosalissa, jossa Adam Riess on aloittamassa kollokvioesitelmää aiheesta.

    Kysymys ja kommentti:

    1) Mistä lähteestä on peräisin tieto, että Hubble kuolemaansa asti kiisti sen, että havainnot osoittavat maailmankaikkeuden laajenevan?

    2) Nykyään Hubblen laki tunnetaan nimellä Hubblen-Lemaîtren laki. Kansainvälinen tähtitieteen unioni päätyi nimeämään lain uudelleen viime syksynä pidetyn jäsenäänestyksen pohjalta (https://www.iau.org/news/pressreleases/detail/iau1812/).

    1. Syksy Räsänen sanoo:

      1) En muista mistä luin siitä alun perin, jostain historiikista. Yhdessä Hubblen viimeisistä artikkeleista toukokuussa 1953 (neljä kuukautta ennen hänen kuolemaansa) Hubble esittää punasiirtymän alkuperän avoimena kysymyksenä, johon pitää löytää vastaus, ja laajenemisen yhtenä mahdollisuutena: https://ui.adsabs.harvard.edu//#abs/1953MNRAS.113..658H/abstract

      Ks. myös Sandagen muistelo: https://apod.nasa.gov/diamond_jubilee/1996/sandage_hubble.html

      Voi kyllä olla, että ilmaisu ”vastustaa” on liian vahva, ”vahvasti epäillä” voisi olla oikeampi.

      2) En tiennytkään tuosta päätöksestä. Miksei ennemmin Lemaîtren-Hubblen laki! Ehkä pitäisi ennemminkin sanoa, että se pitäisi nyt tuntea tuolla nimellä kuin että se tunnetaan sillä nimellä…

      1. Tommi Tenkanen sanoo:

        1) Mielenkiintoista. Sandagen tekstissä kyllä sanotaan mm. ”Hubble must have understood more clearly than anyone what he was dealing with and what he had accomplished.” Tekstin loppupuolella hän tosin pohtii, miksi Hubble ei koskaan tuntunut ottavan kantaa siihen, mitä laajeneminen merkitsee maailmankaikkeuden historian kannalta.

        2) Samaa mieltä molemmista!

        1. Syksy Räsänen sanoo:

          Ajattelin Sandagen tekstin tätä osuutta:

          ”Hubble believed that his count data gave a more reasonable result concerning spatial curvature if the redshift correction was made assuming no recession. To the very end of his writings he maintained this position, favouring (or at the very least keeping open) the model where no true expansion exists, and therefore that the redshift ”represents a hitherto unrecognized principle of nature”. This viewpoint is emphasized (a) in The Realm of the Nebulae, (b) in his reply (Hubble 1937a) to the criticisms of the 1936 papers by Eddington and by McVittie, and (c) in his 1937 Rhodes Lectures published as The Observational Approach to Cosmology (Hubble 1937b). It also persists in his last published scientific paper which is an account of his Darwin Lecture (Hubble 1953).”

  2. Heikki Poroila sanoo:

    Kiitos selkeästä ja maallikollekin avautuvasta analyysistä, jonka kiinnostavin pohdinta ihan lopussa on kuin jännityskertomuksesta.

    Yksi kysymys mittaustapoihin liittyen. Onko punasiirtymän käyttö etäisyyden mittarina kyetty varmentamaan muilla menetelmillä luotettavaksi sekä laajenevan että ei-laajenevan avaruuden oletuksella?

    1. Syksy Räsänen sanoo:

      Punasiirtymä ei suoraan mittaa etäisyyttä, vaan sitä, paljonko maailmankaikkeus on laajentunut. Jos maailmankaikkeus ei laajene, ei ole kosmista punasiirtymääkään. (Paitsi pieni punasiirtymä galaksien paikallisista liikkeistä, esimerkiksi Andromeda tulee meitä kohti, ja siksi sieltä tulevan valo on siirtynyt siniseen päin.)

      Avaruuden laajenemisesta on niin paljon todistusaineistoa, että se on järkevän epäilyn tuolla puolen, vähän niin kuin Maapallon pyöreys.

  3. Ari Juonolainen sanoo:

    Tällaisen maallikon kysymys:

    Laajeneeko tunnettu avaruus meistä poispäin riippumatta missä kohden avaruutta sijaitsemme?

    Havaitsemme meistä kiihtyvällä nopeudella etääntyviä kohteita, mutta havaitsemmeko meitä kohden kiihtyvällä nopeudella lähestyviä kohteita, sillä emme varmastikaan sijaitse big bang nollapisteessä?

    Ja en ole itse lainkaan huolissani vaikka jokin galaxi tulisikin meitä kohden, se tuskin tapahtuu huomenna, juuri silloin kun olen autolla moottoritiellä liikenteessä kohti Tamperetta.

    1. Syksy Räsänen sanoo:

      Laajenee. Ks. https://www.tiede.fi/blogit/maailmankaikkeutta_etsimassa/rajaton_kasvu

      Ei. Mitään big bangin nollapistettä ei ole, se tapahtui samaan aikaan kaikkialla avaruudessa.

      Andromeda on tulossa päin, mutta kestää vielä muutaman miljardi vuotta ennen kuin se tormää Linnunrataan (tai menee läheltä ohi).

  4. mikroi sanoo:

    Pari asiaa hieman aiheen vierestä. Hubblen vakiosta ja sen määrittelystä, sekä siihen liittyvästä tieteen historiasta D.Overbyen Kosmoksen yksinäiset on loistavaa luettavaa, vaikkakin kyseinen opus on jo melko iäkäs.

    Lisäksi Syksy Räsäselle kiitokset loistavasta blogista, erittäin hyvin ja mielenkiintoisesti kirjoitettua tekstiä näistä asioista kiinnostuneille maallikoille!

    Lisäksi jos voi esittää idean blogissa käsiteltäväksi, ehdottaisin tätä viimeisimmistä merkeistä mahdollisesti löytyneistä supersymmetrian hiukkasista Ice Cuben ja ANITAN kokeissa: https://www.youtube.com/watch?v=5ESFGYkkbEI

    1. Syksy Räsänen sanoo:

      Kiitos kiitos.

      En yleensä katso ihmisten lähettämiä videoita, siihen menee niin paljon aikaa.

      Enpä ollutkaan kiinnittänyt huomiota noihin ANITAn tuloksiin, pitää ehkä kirjoittaa niistä jos niiden tulkinnalle uutena hiukkasfysiikkana löytyy tukea.

  5. Mika sanoo:

    Kolmen vaihtoehdon mysteeri -kirjoituksessasi vuodelta 2008 käyt läpi yhtenä vaihtoehtona kiihtyvän laajenemisen selittäjäksi rakenteiden muodostumisen vaikutusta. Onko kuluneina 10 vuotena tästä tullut uutta tietoa, joka vahvistaisi tai heikentäisi kyseistä hypoteesia?

    1. Syksy Räsänen sanoo:

      Nyt tiedetään teoreettisesti paljon paremmin, millä ehdoilla rakenteilla voi olla merkittävä vaikutus maailmankaikkeuden laajenemisnopeuteen, ja ne ovat rajoittavia. Teoreettisesti mahdollisuus ei kuitenkaan ole suljettu pois.

      Havainnoissa ei ole näkynyt mitään poikkeamia tyhjön energiasta, ja jos rakenteiden muodostuminen johtaisi kiihtyvään laajenemiseen, sen vaikutuksen odottaisi olevan siinä määrin erilainen kuin tyhjön energian, että siitä olisi näkynyt merkkejä. Tästäkään asiasta ei kuitenkaan ole varmuutta.

      Hypoteesi on siis heikentynyt.

  6. Lentotaidoton sanoo:

    ”Rakenteiden muodostumisen vaikutus laajenemiseen on pääasiallinen tutkimusaiheeni.
    Rakenteiden vaikutuksen tarkka laskeminen on vaikeaa, eikä vielä tiedetä ovatko ne vastuussa kiihtymisestä, vai tarvitaanko pimeää energiaa tai uutta gravitaatiolakia.”

    Tiedämme männävuosilta tuon tutkimusaiheesi. Viime vuosina et ole kuitenkaan tehnyt (ainakaan näyttäviä) ulostuloja aiheesta. Kaikki blokissasi kirjoitettu on myötäillyt ”standardiselitystä”. Onko huomioni oikea?

    1. Syksy Räsänen sanoo:

      Enpä kai ole tehnyt aiheesta näyttäviä ulostuloja missään vaiheessa.

      En ”myötäile” selitystä tyhjön energiasta, vaan esitän sen (ainakin matemaattisesti) yksinkertaisimpana ja yleisesti suosituimpana, mitä se onkin.

      Yritän olla varovainen siitä, miten esitän omaa tutkimustani blogissa, vaikka olenkin sitä viime aikoina entistä enemmän tehnyt.

  7. Olli S sanoo:

    Kun aika- avaruus laajenee, niin laajeneeko silloin vaan avaruus vai myös aika? Eli onko aika- avaruuden laajeneminen täysin sama asia kuin laajeneminen yleensä vai jotenkin erilainen?

    Onko laajeneminen havainto vai liittyykö se myös siihen että teoriaa pidetään oikeana? Havaintohan on se punasiirtymä.

    1. Syksy Räsänen sanoo:

      Kyse on vain avaruuden laajenemisesta. On olemassa lukuisia havaintoja, joiden perusteella tiedetään, että maailmankaikkeus laajenee. Se on suunnilleen yhtä paljon järkevän epäilyn ulkopuolella kuin se, että Maapallo on pyöreä.

      1. Teero sanoo:

        Jostain suhteellisuusteoriaa käsittelevästä jutusta on muinoin jäänyt päähäni
        ”pythagoralainen” kaarialkiota ds esittävä kaava
        ds⁼ = dx⁼+dy⁼+dz⁼+dT⁼ (jossa nuo ylämerkit tarkoittavat kakkosia), x, y, z ovat tavalliset pituuskoordinaatit ja T on imaginaarinen aikakoordinaati, T = ict (jossa t on tavallinen aika). Tällöin on T:lläkin pituuden dimensio.
        Onko kaavassa mitään järkeä? Kuinka se tulkitaan?

        1. Syksy Räsänen sanoo:

          Menee sen verta kauemmaksi merkinnän aiheesta, että en rupea tätä kommentoimaan.

          Kurssini ”Fysiikka runoilijoille” suppean suhteellisuusteorian osuudesta löytyy tästä muutama sana:

          http://www.courses.physics.helsinki.fi/teor/run/

  8. Täytän tämän kentän hyvin sanoo:

    ”Toinen mahdollisuus on se, että maailmankaikkeuden kiihtyvästä laajenemisesta ei olekaan vastuussa tyhjön energia, joten maailmankaikkeus laajenee eri tavalla kuin mitä odotetaan”

    Maybe

    🤔

  9. Leo Sell sanoo:

    Pari kysymystä: a) Onko kosmologeilla ymmärrystä siitä,miksi KIIHTYVÄ laajeneminen alkoi juuri n.5mrd.v.sitten?

    b) Mikä on pienin laajenemisen yksikkö,vähän niinkuin pienin elämän yksikkö on solu?

    1. Syksy Räsänen sanoo:

      Kiihtyvästä laajenemisesta, ks. sen kohdalla tekstissä olevat linkit:

      http://www.tiede.fi/blogit/maailmankaikkeutta_etsimassa/kirkkaudesta_pimeyteen

      http://www.tiede.fi/blogit/maailmankaikkeutta_etsimassa/kolmen_vaihtoehdon_mysteeri

      Yleisen suhteellisuusteorian mukaan aika-avaruus on jatkuva, siinä ei ole mitään pienintä yksikköä. Aiheesta tarkemmin, ks.

      https://www.ursa.fi/blogi/kosmokseen-kirjoitettua/aika-avaruuden-atomit/

      1. Leo Sell sanoo:

        Kiitos valistus-aiheistasi! a) Eli kiihtyvä laajeneminen olisi voinut alkaa,periaatteessa miltei koska tahansa, koska se riippuu yksinomaan tyhjiöenergiasta (jonka suuruus on yhä arvoitus).

        B) Tarkoitin ”soluilla” universumin tihentymä- ja laajentuma-rakenteita, sellaisia universumin alueita,jotka ovat kyllin massiivisia ja kompleksisia toimiakseen jonkinlaisina autonomisina laajentuma-kiihdyttiminä, kut.”walls and bubbles” (Backreaction Conjuncture).Mutta luin niistä vasta äskettäin tämän koosteen teon yhteydessä.

        1. Syksy Räsänen sanoo:

          A) Niin, jos kiihtyvä laajeneminen johtuu tyhjön energiasta, niin emme tiedä, miksi se on alkanut hiljattain.

          B) Ahaa. Maailmankaikkeus näyttää tilastollisesti samalta suunnilleen 300 miljoonan valovuoden mittakaavalla. Eli jos ottaa laatikon, jonka sivu on vähintään tuon pituinen, niin se laajenee keskimäärin samalla tavalla olipa se missä vain (tilastolliset fluktuaatiot tässä sivuuttaen). Kiihtymisen havaitseminen tuolla etäisyydella on tosin hankalaa, koska sen on niin pientä.

  10. Leo Sell sanoo:

    Nyt ymmärrän paljon paremmin mitä tarkoitetaan ns. avaruuden laakeudella.

    1. Syksy Räsänen sanoo:

      Tässä ei kirjoitettu avaruuden laakeudesta mitään.

      Aiheesta, ks.

      https://www.ursa.fi/blogi/kosmokseen-kirjoitettua/muotoja-ilman-mittanauhaa/

      1. Leo Sell sanoo:

        Selvisi minullekin, että vain BBT:n alkuehdoissa päti kriitiisen massatiheyden hienosäätövaatimus (omegan arvo liki-1) 3-ulotteisen kaareutuneen tilan synnylle, ja että inflatorisen laajenemisen hypoteesi ratkaisi kerralla sekä horisonttiongelman,että LAAKEUS-(flatness) ongelman. Mutta juuri laajetessaan ja ikääntyessään universumi on etääntynyt tuosta arvosta. Eli laajeneminen on ollutkin laakeuden vähenemistä ajan suunnassa! (Mutta tästähän ei tässä yhteydessä ollut kyse,kuten korjasit).

  11. Mika sanoo:

    Kirkkaudesta pimeyteen -merkinnässä kirjoitat näin: ”Tästä päätellään, että maailmankaikkeus on täynnä jotain kummallista ainetta, joka toimii antigravitaation lähteenä, minkä takia kaikki etääntyy toisistaan kiihtyvällä nopeudella. Tälle aineelle on annettu nimi pimeä energia.”

    Merkinnässä puhutaan myös tyhjiön energiasta yhtenä mahdollisena selittäjänä pimeälle energialle.

    Tavallisten ihmisten käsitteistössä energia ja aine ajatellaan usein jollain tavalla eri asioiksi. Olisi kiinnostavaa, jos haluaisit joskus blogissa avata tarkemmin mitä fyysikot tai erityisesti kosmologit tarkoittavat puhuessaan aineesta, joksi ilmeisesti voidaan kutsua myös hiukkasfysiikan kenttiä (näin muistan jollain yleisöluennolla sinun maininneen).

    1. Syksy Räsänen sanoo:

      Kiitos kysymyksestä, mietin. Kyse on lähinnä siitä, että fyysikot käyttävät sanaa ”aine” useissa ristiriitaisissa merkityksissä. Heille on asiayhteydestä selvää, mistä on kyse, mutta ulkopuoliselle sanan käyttö voi olla hämmentävää. (Esimerkiksi juurikin pimeän aineen ja pimeän energian kohdalla.)

  12. Juha sanoo:

    Hei! Onkohan kaikki ”väsyneen valon” teoriayritykset haudattu, vai yrittääkö kukaan nykypäivänä enää keksiä uutta fysikkaa, joka selittäisi punasiirtymää vaihtoehtoisilla tavoilla?

    1. Syksy Räsänen sanoo:

      On haudattu.

      1. pekkap0 sanoo:

        Mihin perustuu varmuus laajenemisesta muiden teorioiden kustannuksella (ja samalla millä perusteella väsynyt valo -teoria on haudattu)?

        1. Syksy Räsänen sanoo:

          Tämä on samanlainen kysymys kuin se, että mihin perustuu varmuus Maapallom pyöreydestä muiden teorioiden kustannuksella.

          Vastauskin on samanlainen: päätelmä perustuu suureen määrään havaintoja ja hyvin ymmärrettyyn teoriaan. Maailmankaikkeuden laajenemisella on keskeinen rooli valon punasiirtymän lisäksi mm. galaksien muodostumisessa ja rakenteessa, atomiydinten synnyssä, atomien synnyssä ja niin edelleen.

  13. Cargo sanoo:

    Jos kaivaa naftaliinista vanhan idean, jonka mukaan maailmankaikkeus on, esim. valonnopeudella laajenevan, neliulotteisen hyperpallon 3D-pinta, niin eikö havaittu kiihtyvä laajeneminen saisi selityksen kun etäisyydet kasvavat ”korkoa korolle” periaatteella?

    Lisäksi, onko jokin erityinen syy miksei tuota näennäisen houkuttelevaa hyperpallo-ideaa pyöritellä mainstream fysiikassa?

    1. Syksy Räsänen sanoo:

      Kysymyksen taustalla vaikuttaa olevan väärinymmärretty idea siitä, että avaruuden kaarevuus liittyisi siihen, että se on upotettu johonkin korkeampiulotteiseen avaruuteen. Näin ei ole.

      Avaruuden kaarevuudesta enemmän täällä: https://www.ursa.fi/blogi/kosmokseen-kirjoitettua/muotoja-ilman-mittanauhaa/

      Tämä riittäköön tästä.

  14. Jesse Ruottu sanoo:

    Täytyy myöntää, että mä vain koitan ymmärtää näitä asioita siinä juurikaan onnistumatta. Mutta tykkään ajatella näitä juttuja. Esimerkiksi, että gravitonilla on antihiukkanen ja se vaikuttaa maailmankaikkeuden laajenemiseen. Jostakin muistan lukeneeni, että tilaa tulee kokoajan lisää ”kaikkialle” kun avaruus laajenee, mutta painovoima pitää galaksit, planeetat ja muut asiat koossa (vai sähkömagneettinen vuorovaikutus vai molemmat) mietin, että ilmestyykö avaruutta jatkuvasti myös maapallon ja meidän kehojen ja talojen kohdalla, mutta me ei vain huomata sitä. Kiitos ja mukavaa kevättä!

    1. Syksy Räsänen sanoo:

      Tämä on sen verta kaukana merkinnän aiheesta, että en kommentoi tarkemmin, mutta avaruuden laajenemisesta enemmän täällä:

      https://www.ursa.fi/blogi/kosmokseen-kirjoitettua/sormustimen-verran/

Vastaa

Sähköpostiosoitettasi ei julkaista. Pakolliset kentät on merkitty *


Kolmesataa

14.3.2019 klo 20.52, kirjoittaja
Kategoriat: Kosmokseen kirjoitettua , Kosmologia

Kollegani Cyril Pitrou vieraili eilen Fysiikan tutkimuslaitoksella Helsingissä puhumassa kevyiden alkuaineiden synnystä. Tämä on ensimmäisiä tutkimuskohteita, joissa kosmologia toi yhteen yleisen suhteellisuusteorian ja hiukkasfysiikan, ja siinä on vieläkin setvimistä.

Tarina alkoi vuonna 1948, kun jatko-opiskelija Ralph Alpher ja hänen ohjaajansa George Gamow julkaisivat reilun sivun mittaisen artikkelin Origin of the Chemical Elements, Alkuaineiden alkuperä. Tunnettu jekkuilija Gamow lisäsi tekijäksi myös fyysikko Hans Bethen, jotta kirjoittajien nimet Alpher-Bethe-Gamow kuulostaisivat englanninkielisen ääntäminä kreikkalaisten aakkosten alulta, alfa-beta-gamma. Bethe ei tiettävästi pannut pahakseen, mutta Alpher ei juuri ilahtunut siitä, että heppoisin perustein mukaan tungettiin arvostettu vanhempi tutkija, jonka hän pelkäsi varjostavan hänen osuuttaan. (Usein muuten jatko-opiskelijat tekevät suurimman osan työstä, en tiedä miten tässä tapauksessa.)

Mitä fysiikkaan tulee, Alpherin ja Gamowin idea oli, että alkuaineet ovat syntyneet protonien ja neutronien törmäillessä toisiinsa varhaisen maailmankaikkeuden kuumassa hiukkaspuurossa. Kun maailmankaikkeus laajenee, aineen tiheys laskee, koska aineen määrä säilyy ja avaruuden tilavuus kasvaa. Toisin sanoen aineen tiheys on sitä isompi, mitä nuorempi maailmankaikkeus on. Vastaavasti aineen lämpötila laskee maailmankaikkeuden laajetessa, eli varhaisempina aikoina aine on kuumempaa.

Kun maailmankaikkeus on alle kahden minuutin ikäinen, protonit ja neutronit ovat vapaita. Ydinvoima yrittää tuoda niitä yhteen, mutta lämpötila on niin iso, että sidos hajoaa saman tien ja hiukkaset menevät omia teitään. Kahden minuutin iässä lämpötila putoaa alle miljardin asteen. Kun protoni ja neutroni kohtaavat ja ydinvoiman vetäminä yhtyvät deuterium-ytimeksi, lämpötila ei enää riitä niiden liiton tuhoamiseen. Deuterium voi sitten törmätä protoniin muodostaen helium-3:a ja niin edelleen.

Alpherin ja Gamowin idea oli, että kaikki alkuaineet syntyvät tällä tapaa, porras kerrallaan raskaampia kohti nousten. Koska lämpötila laskee koko ajan, ydinten muodostumiseen on kuitenkin käytössä vain lyhyt aika.

Helium-4:n muodostuminen loppuu noin viiden minuutin iässä, kun kaikki neutronit on käytetty. Raskaampia alkuaineita ei ehdi muodostua kuin vähäisiä määriä, koska maailmankaikkeudesta tulee niin kylmä että ydinreaktiot sammuvat. Cyril huomauttikin, että nobelisti Steven Weinbergin menestyskirjan Ensimmäiset kolme minuuttia nimen olisi pitänyt olla Ensimmäiset kolmesataa sekuntia.

Tuloksena on keitos, josta 76% on vedyn isotooppeja (yksinäisiä protoneita ja deuteriumia) ja 24% heliumin isotooppeja (helium-3 ja helium-4, joissa on yksi tai kaksi neutronia). Raskaampia alkuaineita syntyy vain vähän, eniten -miljardisosan verran- litium-7:ää (jossa on kolme protonia ja neljä neutronia). Raskaammat alkuaineet kehittyvät vuosimiljoonia myöhemmin tähtien fuusioreaktioissa, räjähdyksissä ja törmäyksissä.

Kevyiden alkuaineiden synty tuo yhteen yleisen suhteellisuusteorian ja hiukkasfysiikan Standardimallin kaikki vuorovaikutukset. Gravitaatio kertoo, miten maailmankaikkeus laajenee ja jäähtyy. Ydinvoima, joka on vahvan vuorovaikutuksen jäännös, vetää protoneita ja neutroneita yhteen, kun taas sähkömagneettinen vuorovaikutus työntää protoneita erilleen: näiden kahden voiman kilpailu määrää sen, millaisia ytimiä syntyy. Heikko vuorovaikutus taasen kertoo, miten ytimet hajoavat muodostuttuaan.

Kaikki nämä vuorovaikutukset on mitattu Maapallolla ja Aurinkokunnassa nykypäivänä. Ainoa tuntematon seikka on se, paljonko säteilyä (fotoneita ja neutriinoja) alkuminuuteilla on suhteessa protoneihin ja neutroneihin – eli kuinka kuuma maailmankaikkeus on. (Nykyään tämä voidaan määrittää erikseen kosmisesta mikroaaltotaustasta.) Niinpä teoria on ennustusvoimainen: tuntemattomia lukuja on yksi ja ennusteita on neljä: deuteriumin, helium-3:n, helium-4:n ja litium-7:n määrä suhteessa yksin jääneisiin protoneihin.

Kolmen ensimmäisen ytimen osalta ennustukset vastaavat havaintoja. Tämä on merkittävä saavutus: tulos osoittaa, että nykyään mittaamamme luonnonlait pätivät maailmankaikkeuden alkuminuutteina. Sittemmin luonnonlakeja on luodattu kauas ensimmäisen sekunnin perukoille kosmisen inflaation kautta, mutta kevyiden alkuaineiden synty oli ensimmäinen onnistunut katsaus maailmankaikkeuden alkuhetkiin, ja osoitti miten varhaista maailmankaikkeutta voi käyttää hiukkasfysiikan laboratoriona.

Litium-7:n kohdalla ennustus menee kuitenkin pieleen. Sitä havaitaan vain kolmannes ennustetusta määrästä. Joitakin vuosia helpoin selitys oli mittausvirheet siinä, miten voimakkaasti ytimet vuorovaikuttavat toistensa kanssa. Kun virherajat ovat kutistuneet, ero ennustuksen ja havaintojen välillä on kuitenkin vain kasvanut. Nykyään ydinfysiikka tunnetaan niin hyvin, että sitä on vaikea sorkkia pilaamatta onnistuneita ennusteita muille ytimille.

Cyril arveli, että vika piilee tähtien mallintamisessa. Litiumin määrä nimittäin mitataan vanhojen tähtien pinnalta. Jos tähdissä kiertää arveltua enemmän litiumia pinnalta keskustaan, se voi palaa siellä pois. Tähtien fysiikan asiantuntijoiden on kuitenkin ollut vaikea ymmärtää, miten kaksi kolmannesta litium-7:stä voisi tuhoutua siten, että muiden ydinten kohdalla ei tule ongelmia. Lisäksi mekanismin pitäisi toimia samalla tavalla monissa erilaisissa tähdissä, joissa näkyy suunnilleen sama määrä litium-7:ää.

Litium-ongelma voi myös olla merkki jostain Standardimallin tuonpuoleisesta fysiikasta, joka tuhoaa litiumia. On esimerkiksi ehdotettu, että on olemassa hieman pimeän aineen hiukkasta raskaampi hiukkanen, joka hajoaa pimeäksi aineeksi ja fotoniksi. Jos hajoamisessa syntyvän fotonin energia on juuri sopiva litium-7-ytimen rikkomiseen, se voi hävittää niitä juuri sopivasti. Pitää taas olla huolellinen, että samalla ei riko muita ytimiä, mutta tämä onnistuu.

Ei tiedetä mistä suunnasta ratkaisu löytyy, mutta on selvää, että vastauksen löytämiseksi pitää tuntea erilaisia fysiikan haaroja, yleisesti suhteellisuusteoriasta vanhojen tähtien rakenteeseen.

17 kommenttia “Kolmesataa”

  1. Anne Liljeström sanoo:

    Tunnettu jekkuilija! <3

    1. Kuten käytännön pila nimeltä Gamowin tekijä, joka hämää plasmafuusioihmisiä yhä !

  2. Lentotaidoton sanoo:

    ”Litium-ongelma voi myös olla merkki jostain Standardimallin tuonpuoleisesta fysiikasta, joka tuhoaa litiumia”.

    Kaikkea on kokeltu/tutkittu mutta probleema tuntuu pysyvän (neutronisieppauksetkaan CERNissä eivät tuottaneet vastausta). Voisivatko massiiviset neutriinot olla yksi ratkaisukeino?

    1. Syksy Räsänen sanoo:

      Miten ajattelet massiivisten neutriinoiden liittyvän asiaan?

  3. Lentotaidoton sanoo:

    https://en.wikipedia.org/wiki/Big_Bang_nucleosynthesis

    These pieces of additional physics include relaxing or removing the assumption of homogeneity, or inserting new particles such as massive neutrinos.
    The second reason for researching non-standard BBN, and largely the focus of non-standard BBN in the early 21st century, is to use BBN to place limits on unknown or speculative physics. For example, standard BBN assumes that no exotic hypothetical particles were involved in BBN. One can insert a hypothetical particle (such as a massive neutrino) and see what has to happen before BBN predicts abundances that are very different from observations. This has been done to put limits on the mass of a stable tau neutrino.

    https://arxiv.org/pdf/1412.1408.pdf

    Moreover, SUSY decays can destroy 7Li and/or produce 6Li, leading to a possible supersymmetric solution to the lithium problems noted above

    1. Syksy Räsänen sanoo:

      Ilmeisesti tässä haetaan sitä, että steriilien neutriinoiden oskillaatio aktiivisiksi neutriinoiksi (ja toisin päin) vaikuttaisi ydinteen syntyyn. esim. muuttamalla elektronin neutriinoiden määrää. En ole törmännyt malleihin, joissa litium-ongelmaa yritettäisiin selittää tällä tavalla.

      Neutriinot ovat lakanneet vuorovaikuttamasta muun aineen kanssa jo ennen ydinten synnyn alkua. Kun neutriinoja ei enää ole käytettävissä, protonit eivät voi enää muuttua neutroneiksi, neutronit vain hajoavat protoneiksi. Niinpä aikaväli neutriinoiden irtikytkeytymisen ja deuteriumin synnyn välillä määrää sen, paljonko neutroneita on käytössä, millä on iso vaikutus siihen, millaisia ytimiä syntyy. En kuitenkaan ole kuullut, että sitä säätämällä voisi vähentää litiumin pitoisuutta kolmannekseen pilaamatta muiden ydinten tarkkaan mitattuja pitoisuuksia.

      Wikipedian fysiikka-artikkelien laatu vaihtelee, en itse käyttäisi niitä lähteenä. (Sen sijaan monien matematiikka-artikkeleiden taso vaikuttaa hyvältä.)

  4. Erkki Kolehmainen sanoo:

    NMR-spktroskopiaa vuosikymmeniä tehneenä jäin ihmettelemään, miksei synny Li-6- ja 3-H-ytimiä. Entä 9-Be?

    1. Syksy Räsänen sanoo:

      Melkein kaikki tritium kuluu He-4:n valmistamiseen, prosesseissa H-3 + p -> He-4 ja H-3 + H-2 -> He-4 + n. Vaikka sitä jäisi aluksi yli, niin tritium hajoaa 12 vuoden puoliintumisajalla He-3:ksi.

      Litium-6:sta syntyy, mutta sitä myös hajoaa törmäyksissä, koska sen sidosenergia on pieni. Luulen että sen takia sen loppupitoisuus on pieni.

      Beryllium-7 syntyy niin myöhään, että vapaita neutroneita ei enää juuri ole Be-9:n tuottamiseksi (ja lämpötila on myös laskenut). Tuotettu Be-7 sitten hajoaa Li-7:ksi.

      1. Leo Sell sanoo:

        Vieläkö aikaikkunasi on auki? Fotoniepookin aikaisessa (10s.- 20min) universumissa,jossa vallitsi protonien elektronien ja fotonien plasmatila on Litiumin määrä arvioitu ideaalikaasuoletusten mukaisesti. On esitetty myös oletus että ”non-extensive-statistics”- mallinnus toisi esille/helium -beryllium-litium /-helium/ -vuorovaikutusten hierarkisia suhteita. Tällöin beryllium(7)tasot putoavat ja kuten myös litiumin(6), ja litium(8)-litium(9) hajonta takaisin heliumiksi(6) kasvaa odotettua enemmän – voitaisiin päästä jopa tuohon 1/3-osan löydökseen. Artikkeli ”The universe have a lithium problem”, Evan Gough,20.2.2017 (New paper from researches in China)

        1. Syksy Räsänen sanoo:

          En tiedä mitä tuo tarkoittaa.

          1. Leo Sell sanoo:

            Se taitaa tarkoittaa,että BBT:ssä Maxwell-Bolzman-jakautuma kuvaa,mitä tapahtuu ideaalikaasuolosuhteissa,ei vältttämättä tapahdu reaalikaasuilla noissa ääriolosuhteissa. Koska Li-7 isotooppia syntyy vain Be-7-isotoopista,niin mikäli Be-7 isotooppia alun nukleosynteesissä syntyykin odotettua vähemmän,syntyy myös litiumia vähemmän. Ja edelleen,että Li-7-isotoopin takaisin Helium-4:ksi muuttumisen määrä pp2-kejuissa voi olla odotusarvoja isompi. En ole asiantuntija, enkä tiedä onko tämä mahdollista,olen vain asiasta kiinnostunut.

  5. Voisi kuvitella että riittävän suuri määrä neutriinoja olisi voinut vähän kiihdyttää vapaan neutronin betahajoamista protoniksi ja elektroniksi. Eli kun normaalisti prosessi on n->p+e+antinue, niin sama verteksi on myös nue+n->p+e. Kosmiset neutriinot ovat nykyään niin matalaenergisiä että ne eivät näy mittauksissa, joten niiden tiheyttä ei suoraan pysty mittaamaan. Varmaankin tämän on joku laskenut, että onko efektillä ollut jotain merkitystä nukleosynteesiin vai ei.

    1. Syksy Räsänen sanoo:

      Kosmisten neutriinoiden määrä vaikuttaa laajenemisnopeuteen sekä ydinten synnyn että mikroaaltotaustan aikaan. Tämä on niiden pääasiallinen vaikutus ydinten syntyyn. Niitä voi olla vain 15% tavallista enemmän, muuten laajeneminen on liian nopeaa, neutroneita on jäljellä liian paljon ja helium-4:ää syntyy liikaa.

      Tuo mainitsemasi reaktio ei enää juuri vaikuta ydinten synnyn aikaan, koska neutriinojen vuorovaikutukset ovat niin heikkoja ydinten synnyn lämpötilassa, että ne eivät juuri törmäile neutroneihin. Standardineutriinojen korjaus prosessin kulkuun on vain 1.5%. Niiden lukumäärän kasvattaminen 15%:lla on siis vain pieni korjaus pieneen korjaukseen, eikä käsittääkseni voi pudottaa litiumin osuutta kolmannekseen.

  6. Irina Armfelt sanoo:

    Eihän maailmakaikkeus voi olla olla loputon. Jossainhan on pakko olla ”reuna” tai seinä johon avaruus päättyy. Jos avaruus päättyy johonkin niin mitä sen jälkeen on? Tai jos ei pääty niin miten avaruus voi aina vain jatkua ja jatkua. Enni 6 v puolesta kysyy famu Irina

    1. Syksy Räsänen sanoo:

      Avaruus voi olla ääretön.

      Jos avaruus ei ole ääretön, sillä ei silti ole reunaa. Esimerkiksi pallon pinta on äärellinen, mutta sillä ei ole reunaa.

      Lisää aiheesta täällä:

      https://www.ursa.fi/blogi/kosmokseen-kirjoitettua/muotoja-ilman-mittanauhaa/

      Ei tästä tämän enempää, kun ei liity merkinnän aiheeseen.

  7. Leo Sell sanoo:

    Vielä korjaus edelliseen,Beryllium-7 hajoaa tietenkin Litium-7:ksi (ei 6-eksi,kuten taisin edellä kirjoittaa.

Vastaa

Sähköpostiosoitettasi ei julkaista. Pakolliset kentät on merkitty *


Parempi väärässä kuin sekaisin

22.2.2019 klo 21.43, kirjoittaja
Kategoriat: Kosmokseen kirjoitettua , Kosmologia

Edellisessä merkinnässä mainitsin, että idea kosmisesta inflaatiosta syntyi hiukkasfysiikan yhtenäisteorioiden piirissä. Kyse on nimenomaisesti suurista yhtenäisteorioista, Grand Unified Theories eli GUTs. (Alkuperäiselle hengelle uskollisin suomennos lyhenteelle olisi varmaan SISU – joku keksinee sopivat sanat joista tämä saataisiin.) Ne ovat hiukkasfysiikan Standardimallin yläpuolella viimeinen askelma ennen kvanttigravitaation ja kaiken teorian tuomista mukaan kuvioihin.

Suurten yhtenäisteorioiden juuret juontavat 70-luvulle. Silloin oli saatu valmiiksi hiukkasfysiikan Standardimalli, joka kattoi sähkömagneettisen vuorovaikutuksen sekä heikon ja vahvan vuorovaikutuksen. Standardimalli osoitti, että sähkömagneettinen ja heikko vuorovaikutus ovat vain osia sähköheikosta vuorovaikutuksesta. Ne näyttävät erilaisilta, koska Higgsin kenttä antaa massan heikkoa vuorovaikutusta välittäville W– ja Z-bosoneille, mutta ei sähkömagneettista vuorovaikutusta välittäville fotoneille.

Mitä isompi massa hiukkasella on, sitä lyhyempi kantama sen välittämällä vuorovaikutuksella on. Massaa alemmilla alhaisemmilla energioilla vuorovaikutus on myös sitä heikompi, mitä isompi välittäjän massa on. Korkeilla energioilla, jotka LHC-kiihdyttimessä saavutetaan, heikko vuorovaikutus on yhtä voimakas kuin sähkömagneettinenkin. Varhaisen maailmankaikkeuden (ennen 10^(-11) sekuntia) korkeissa lämpötiloissa (yli miljoona miljardia astetta) Higgsin kenttä on sulanut eikä anna hiukkasille massoja, joten silloinkin sähkömagneettinen ja heikko vuorovaikutus yhtyvät.

Standardimallissa vahva vuorovaikutus on kuitenkin erillinen kokonaisuus, joka on vain pultattu kiinni sähköheikkoon vuorovaikutukseen. Suurten yhtenäisteorioiden lähtökohta on yksinkertainen: entäpä jos vielä suuremmilla energioilla sähköheikko ja vahva vuorovaikutuskin yhtyvät?

Suuren yhtenäisteorian voi rakentaa samalla reseptillä kuin sähköheikon vuorovaikutuksen. Kirjoitetaan paperille sellainen vuorovaikutus, jossa on tarpeeksi monta palikkaa selittämään sekä sähköheikkoa vuorovaikutusta välittävät kolme hiukkasta että vahvaa vuorovaikutusta välittävät kahdeksan gluonia. Kun sähköheikko vuorovaikutus ja vahva vuorovaikutus yhdistetään, mukaan tulee väistämättä tulee uusia hiukkasia, jotka välittävät niitä molempia samaan aikaan. Nämä hiukkaset on nimetty mielikuvituksellisesti X– ja Y-bosoneiksi. Koska tällaisia hiukkasia ei ole havaittu, teoriaan pannaan uusi Higgsin kenttä, joka antaa niille korkean massan. Tämä rikkoo sähköheikon ja vahvan vuorovaikutuksen yhtenäisyyden ja työntää uudet hiukkaset korkeille energioille.

Koska suuren yhtenäisteorian uudet välittäjähiukkaset välittävät sekä sähköheikkoa että vahvaa vuorovaikutusta, ne eivät erottele Standardimallin hiukkasia sen mukaan, tuntevatko ne vahvan vuorovaikutuksen vaiko eivät, vaan tarttuvat kiinni kaikkiin. Standardimallissa vain kvarkit tuntevat vahvan vuorovaikutuksen, mikä takaa sen, että protoni, joka koostuu kolmesta kvarkista, on vakaa:  se koostuu kolmesta kevyimmästä kvarkista, joten sillä ei ole mitään, mihin hajota. Sen sijaan suuressa yhtenäisteoriassa kvarkit voivat muuttua elektroneiksi ja muiksi hiukkasiksi, minkä myötä protoni voi hajota. Jotta hajoamista tapahtuisi tarpeeksi harvoin, välittäjähiukkasten massan pitää olla hyvin iso, noin 10^(14) kertaa W– ja Z-bosonien massa.

Samaan korkeaan massaan päädyttiin toistakin reittiä. 1970-80-luvulla vaikutti siltä, että Standardimallin sähkömagneettisen, heikon ja vahvan vuorovaikutuksen voimakkuudet yhtyvät tällä samalla energiaskaalalla. (Vuorovaikutusten voimakkuus nimittäin riippuu energiasta.) Jos kahden riippumattoman polun päätyminen samaan paikkaan ei olisi riittänyt, niin kosmologia antoi kolmannen vihjeen siitä, että ollaan oikeilla jäljillä.

Sähköheikon ja vahvan vuorovaikutuksen eroon liittyy Higgsin kentän olomuodon muutos. 70-luvulla Tom Kibble hahmotti, että siinä syntyy kosmisia säikeitä ja muita virheitä Higgsin kentässä, hieman kuin veden jäätyessä. Näiden kosmisten säikeiden ehdotettiin sekoittavan maailmankaikkeuden ainetta ja synnyttävän siten rakenteen siemenet. Säikeiden vispaamien epätasaisuuksien koko riippuu niiden jännityksestä, mikä liittyy suoraan uusien välittäjähiukkasten massaan. Protonien hajoamisesta ja vuorovaikutusten yhdistymisesta päätelty energia antoi juuri oikean suuruuden, joka sopi havaintoihin.

Kolme aivan erilaista argumenttia johtivat samaan kauniiseen tuloksen. Sääli vain, että ne olivat kaikki väärin.

Ensinnäkin, protonin hajoamista ei ole nähty. Koska suuren yhtenäisteorian vuorovaikutus on heikko, protonin hajoaminen on harvinaista. Tämän ongelman voi ratkaista tarkkailemalla montaa protonia. Japanilainen koe Kamiokande tarttui haasteeseen. Vuoren sisälle Kamiokan kaivokseen rakennettiin vuosina 1982-83 kolmen miljoonan litran vesisäiliö, jonka reunat vuorattiin valoa havaitsevilla laitteilla. Protonin hajoamisesta seuraa valonvälähdys, ja tankissa oli noin 10^(33) protonia. Sitten tarvitsi vain odottaa.

Sen enempää Kamiokande kuin sen vuonna 1996 viisitoista kertaa isommaksi laajennettu versio Super-Kamiokande eivät nähneet yhdenkään protonin hajoavan. Tästä tiedetään, että protonin elinikä on vähintään 10^(33) vuotta. Tämä on satatuhatta miljardia miljardia kertaa maailmankaikkeuden ikä. Protonin ikää voi kasvattaa nostamalla sitä hajottavan välittäjähiukkasen massaa, mutta ei mielin määrin. Yksinkertaisimmassa suuressa yhtenäisteoriassa oli massalle vain pieni ikkuna – mikä olikin motivaatio Kamiokandelle ja sen superversiolle. Kamiokande ja Super-Kamiokande sulkivat tämän ikkunan: yksinkertaisin suuri yhtenäisteoria ei kuvaa todellisuutta.

Toisekseen, mitä vuorovaikutusten yhtymiseen tulee, tarkemmat mittaukset ovat osoittaneet, että Standardimallissa niiden voimakkuudet eivät kohtaakaan odotetulla yhtenäisteorian energialla, tai itse asiassa missään. Tämä ei ole ratkaiseva muutos, koska yhtenäisteoriassa vuorovaikutukset voivat kehittyä eri tavalla kuin Standardimallissa jo ennen yhtymistään, mutta murentaa motivaatiota.

Kolmannekseen, 90-luvun havainnot kosmisesta mikroaaltotaustasta osoittivat, että inflaation ennustukset rakenteen siemenistä pitävät paikkansa ja kosmisten säikeiden ennustukset eivät pidä paikkaansa.

Havainnot sulkevat pois vain suurten yhtenäisteorioiden yksinkertaisimmat versiot, mikään ei estä rakentamasta monimutkaisempia teorioita. Vaatimuksena on vain se, että yhtenäisvuorovaikutus pitää sisällään sähköheikon ja vahvan vuorovaikutuksen. Yksinkertaisinta on ottaa sellainen vuorovaikutus, joka lisää mahdollisimman vähän uusia hiukkasia. Ei kuitenkaan ole mitään rajaa sille, miten monimutkaisen elämästään voi tehdä, ja sama pätee teorioihin: vuorovaikutuksia lisäämällä voi työntää protonin elinikää korkeammalle. Apuun on otettu myös supersymmetria, joka sopivasti sovellettuna auttaa estämään protonin hajoamista.

Epäonnistumiselle voi keksiä monia selityksiä, mutta harvoin ne ovat yhtä vakuuttavia kuin onnistuminen. Todisteiden puute sai tutkijat arvioimaan suuria yhtenäisteorioita kriittisemmin. Alusta asti oli hankala ymmärtää, miksi X– ja Y-bosonien massa on niin valtavan paljon isompi kuin W– ja Z-bosonien massa.

Yhtenäisteoriassa kaikki välittäjähiukkaset käyttäytyvät korkeilla energioilla samalla tavalla, ja ero tulee niiden erilaisesta kytkennästä Higgsin kenttiin. Standardimallin Higgs antaa pienen massan, uusi Higgs ison. Mutta miksi? Yhtenäisteoriassa nimittäin molemmat Higgsit ovat aluksi osa samaa kokonaisuutta, yhtenäistä Higgsin kenttää. Teorian matemaattinen rakenne ei kiellä sitä, että Higgsin eri osat käyttäytyvät eri tavalla, mutta se on vähintäänkin teorian hengen vastaista – sanalla sanoen rumaa. Ottaen huomioon, että yksi tärkeimpiä motivaatioita suurille yhtenäisteorioille oli samanlaisen estetiikan seuraaminen, joka oli vienyt voittoon Standardimallin kanssa, esteettiset ongelmat ovat vakava asia.

Mutta vaikka osoittautuisi, että suurta yhtenäisteoriaa ei ole, olisi yksisilmäistä tuomita sen tutkiminen harhapoluksi.

Kuten kosmiset säikeet, inflaatio kehittyi suurten yhtenäisteorioiden kehdossa. Alun perin yksi inflaation motivaatio oli sen selittäminen, miksi suuren yhtenäisteorian olomuodon muutoksissa syntyviä virheitä Higgsin kentässä (kosmisia säikeitä ja muita) ei näy. Inflaation haluttiin pyyhkivän ne pois, ja suurin osa varhaisista inflaatiomalleista perustui suuren yhtenäisteorian olomuodon muutokseen. Sittemmin nämä rakennustelineet on voitu purkaa, eikä inflaatio kaipaa suurta yhtenäisteoriaa. Kiinnittyminen yhtenäisteorian ideaan saattoi kyllä vaikuttaa siihen, että kesti 27 vuotta huomata, että Standardimallin Higgskin voi olla vastuussa inflaatiosta: katse oli suunnattu liian korkealle.

Mitä kokeelliseen puoleen tulee, Super-Kamiokande ei nähnyt protonin hajoamista, mutta se havaitsi neutriinojen muuttumisen toisikseen. Löytö oli ensimmäinen varma merkki Standardimallin tuonpuoleisesta fysiikasta, ja siitä myönnettiin Nobelin palkinto vuonna 2015. Super-Kamiokandesta on ollut muutakin hyötyä (pieni esimerkki: se on rajoittanut heksakvarkkien mahdollisuutta) ja samaan vuorenalaiseen laboratorioon on rakennettu myös gravitaatioaaltokoe KAGRA.

Tutkimuksen polut ovat harvoin suoria. Suurten yhtenäisteorioiden tapaus osoittaa, että selvältä vaikuttavat vihjeet voivat viedä suohon. Mutta siitä näkee myös sen, miten väärät ideat voivat ohjata tärkeisiin löytöihin, kun ne keskittävät ajattelua hedelmälliseen suuntaan. Prototieteilijä Francis Baconin sanoin, ”totuus kehkeytyy ennemmin virheestä kuin sekasorrosta”.

15 kommenttia “Parempi väärässä kuin sekaisin”

  1. Eusa sanoo:

    ”vuorenalaisessa” jäänyt inessiiviin.

    SISU = Standardimallin Itujen Suuri Uute. 🙂

    1. Syksy Räsänen sanoo:

      Kiitos, korjasin.

  2. Anekdootti suuresta yhtenäisteemasta. Kun olin jatko-opiskelijana 1990-luvun alussa, EISCAT-tutkan data-analyysiin olivat jotkut kehittäneet kasan matlab-skriptejä, mikä uskokaa tai älkää oli silloin uusi kova juttu. Ohjelmistolle he antoivat nimen GUISDAP, Grand Unified Incoherent Scatter Data Analysis Program. Muutamia vuosia myöhemmin tein MHD-simulaatiota, ja vitsin vuoksi annoin koodille nimeksi GUMICS, Grand Unified Magnetosphere-Ionosphere Coupling Simulation. GUMICS on käytössä edelleen, ja siitä tulikin mieleeni että pitäisi nykiä osastopäällikköä hihasta että hän ehtisi paneutua sen lisenssiasiaan, jotta eräs HY:n ryhmäkin saisi siitä hyötyä omaan sovellukseensa. Taitaa olla luonnonlaki että suuret yhtenäisasiat lopulta hajoavat ja murenevat byrokratiaksi.

  3. Onko positronin ja protonin varauksien tarkalle yhtäläisyydelle muita selityksiä kuin GUT?

    1. Syksy Räsänen sanoo:

      GUTteja tosiaan joskus mainostetaan sillä, että ne sitovat kvarkkien ja leptonien varaukset yhteen.

      Tämä on kuitenkin totta jo Standardimallissa. Anomalioiden puute asettaa ehtoja sähkövarauksille, ne eivät voi olla mielivaltaisia.

  4. Cargo sanoo:

    Kumpihan on vaikeampaa: kehittää toimiva GUT vaiko toimiva ilmastomalli?

    Median hysterisoima ilmastoahdistus takaa tutkimusrahoituksen kestävyyden perseelleen menevälle pelottelulle, mutta mikä onkaan teoreettisen fysiikan tutkimusrahoituksen tulevaisuus, jos siis mitään toimivaa ei pikkuhiljaa ala löytymään?

    Peukalot kääntyvät alaspäin ja työhuoneet luovutetaan esim. sukupuolentutkijoille 🙁

    1. Syksy Räsänen sanoo:

      Nämä ovat aivan erilaisia kysymyksiä.

      Suuressa yhtenäisteoriassa on kyse siitä, onko todellisuudessa olemassa sellaista fysiikkaa, jota se kuvaa, eli perustavanlaatuisten lakien löytämisestä.

      Ilmaston ja ilmastonmuutoksen mallintamisessa on kyse tunnetun fysiikan soveltamisesta monimutkaiseen systeemiin riittävän tarkasti.

      Onneksi ilmastotutkijoiden vuosikymmeniä esitetyt varoitukset otetaan viimein julkisuudessa todesta, ilmastonmuutos kun on eräs suurimpia ihmiskuntaa vastassa olevia uhkia.

      Mitä suurten yhtenäisteorioiden tutkimisen hedelmällisyyteen tulee, kuten merkinnässä kirjoitin, niistä kumpusi inflaatio, joka on ennustanut havaintoja erittäin onnistuneesti.

  5. Standardimallin 18-26 parametrista (https://spinor.info/weblog/?p=6355) suurin osa liittyy generaatioiden heikkoon kytkentään, ja vain yksi parametri liittyy vahvaan vuorovaikutukseen, eli sen kytkentävoimakkuus g3. Onko yritetty sellaisia sähköheikon teorian yleistyksiä jotka pyrkisivät selittämään generaatioiden lukumäärän ja niiden heikot kytkennät, mutta jättävät vahvan vuorovaikutuksen teorian ulkopuolelle? Motivaationa siis olisi parametrien vähentämisen ekonomia.

    1. Syksy Räsänen sanoo:

      Ei parametreja noin voi laskea. Suurin osa parametreista liittyy massoihin ja kvarkkien sekoittumiseen. Heikkon vuorovaikutuksen kytkentään liittyy kaksi parametria, vahvaan yksi.

      En tiedä, onko mitään malleja, missä tutkittaisiin sähköheikkoa vuorovaikutusta ilman vahvaa ja samalla sukupolvien määrää – luultavasti, kaikenlaista kun on tehty.

      1. Kyllä, mutta tarkoitin siis liittää ne sekoitusparametrit sähköheikkoon, koska se toinen vaihtoehto (vahva vuorovaikutus) ei kytke sukupolvia toisiinsa.

        1. Syksy Räsänen sanoo:

          Onhan se niinkin. (Sähköheikko vuorovaikutushan on siis konsistentti minkä tahansa sukupolvien määrän kanssa, sille ei ole mitään teoreettista rajoitusta.)

  6. Lentotaidoton sanoo:

    Mehän tiedämme yhden standarditeorian suuren vaikeuden. Se ei selitä baryonigeneesiä, eli miksi ainetta on enemmän kuin antiainetta. Tai selittää mutta heikosti. Esim kokeissa CP rikko sähköheikkovoimassa up to 13% (voi olla nykyään jo isompikin). Myös kromodynamiikassa tulisi olla vastaava CP violaatio, sitä ei kuitenkaan ole nähty (vahva CP-ongelma).

    Saharovin ehdot: baryoniluvun muuttuminen, CR ja CPR symmetrian rikkoutuminen ja poikkeama termisestä tasapainosta. Kun standarditeoria selittää nämä vain heikosti, niin aikoinaan toivo laitettiin näiden superraskaiden X ja Y bosonien hajoamiseen ja siten baryoni- ja leptonilukujen muuttumiseen.

    Toisaalta kysymysmerkki on miksi nykyisen Higgsin kentän arvo on mitätön vaadittuihin muiden Higgsin kenttien arvoon 10^16 GeV, eli hierarkiaongelma. Voitko valaista näitä lisää?

    1. Syksy Räsänen sanoo:

      Tässä onkin useita asioita.

      Hierarkiaongelma on suurten yhtenäisteorioiden aiheuttama ongelma. Suosituin yritys sen ratkaisemiseen on supersymmetria, joka rajaa Higgsin massaan tulevan korjauksen supersymmetrian rikkoutumisskaalaan. Tämä ei kuitenkaan selitä sitä, miksi alun perin teoriassa on kaksi eri skaalaa eri Higgseille.

      Baryogeneesiin on tosiaan suurissa yhtenäisteorioissa paremmat ainekset. (Itse asiassa Standardimallissakin sähköheikkoa skaalaa korkeammilla lämpötiloilla baryoniluku ei säily, kaksi muuta ehtoa vain valitettavasti eivät toteudu tarpeeksi vahvasti.)

  7. Hannu Piesanen sanoo:

    SISU = SamperinIsoSepustusUniversumista

  8. Lentotaidoton sanoo:

    SISU = SotemainenInnokkuusSorsiaUudistuksia

Vastaa

Sähköpostiosoitettasi ei julkaista. Pakolliset kentät on merkitty *


Ylös pohjalta

14.2.2019 klo 19.19, kirjoittaja
Kategoriat: Kosmokseen kirjoitettua , Kosmologia

Merkinnässä Alas huipulta kirjoitin fysiikan teorioiden kehityksestä kohti matemaattisesti hienostuneempia korkeuksia. Hiukkasfysiikassa on kuitenkin kaksi lähestymistapaa teorioihin: top-down ja bottom-up. Edellisessä kasataan hyvin yleinen teoria matemaattisista ja esteettisistä periaatteista lähtien ja katsotaan, millaisia seurauksia sillä on. Jälkimmäisessä lähdetään tunnettujen asioiden tienoilta, yritetään ratkaista rajattuja ongelmia ja edetään askel kerrallaan.

Tunnetuin esimerkki onnistuneesta huipulta löytyneestä teoriasta on yleinen suhteellisuusteoria. Havainnoilla oli sen muotoilussa vain vähäinen rooli, sen sijaan Albert Einstein, David Hilbert, Michele Besso ja Marcel Grossmann perustivat työnsä filosofisiin ja matemaattisiin pohdintoihin. Yleistä suhteellisuusteoriaa on testattu tarkkaan, ja se on yli sadan vuoden ajan ennustanut tarkasti ilmiöitä, joista ei sitä kehitettäessä ollut vielä aavistustakaan, GPS-satelliittien liikkeistä maailmankaikkeuden laajenemiseen ja mustien aukkojen törmäyksistä syntyviin gravitaatioaaltoihin. Einsteinin ja Hilbertin vuonna 1915 esittämästä yleisen suhteellisuusteorian muotoilusta ei ole tarvinnut yli sadan vuoden aikana muuttaa piiruakaan – ellei sitten kiihtyvään laajenemiseen liittyvän kosmologisen vakion osalta.

Tämä on poikkeuksellista. Kosminen inflaatio tarjoaa tyypillisemmän esimerkin teorioiden kehityksestä. Sen lähtökohtana oli yksinkertainen ongelma: miksi maailmankaikkeus näyttää samanlaiselta joka suunnassa? Koska valo kulkee äärellisellä nopeudella ja maailmankaikkeus on äärellisen ikäinen, vaikuttaisi siltä, että kaukana toisistaan olevat alueet eivät ole ehtineet olla kosketuksissa. Miten ne siis tietävät olla samanlaisia?

Ongelmaa yritettiin 1960-luvulta lähtien ratkaista yleisen suhteellisuusteorian parissa niin kutsutulla Mixmaster-mallilla (suomeksi siis tehosekoittimella), jossa eri alueet sekoittuvat varhaisina aikoina. Nämä yritykset eivät johtaneet tyydyttävään selitykseen, ja vastaus löytyi eri suunnasta.

Vuonna 1980 Aleksei Starobinsky esitti, että kvanttifysiikan korjaukset yleiseen suhteellisuusteoriaan johtavat kiihtyvään laajenemiseen varhaisina aikoina. Starobinsky ei yhdistänyt malliaan kysymykseen sitä, miksi maailmankaikkeus näyttää samalta kaikkialla. Mutta heti samana vuonna Demosthenes Kazanas selitti, miten kiihtyvä laajeneminen voisi ratkaista ongelman.

Kazanas esitti, että jossain hiukkasfysiikan yhtenäisteoriassa on Higgsin kentän kaltainen kenttä, jonka tyhjiön energiaan liittyvä gravitaatio on hylkivää. Tämä saa aikaan kiihtyvän laajenemisen ja avaruuden osat työntyvät pois toisistaan niin nopeasti, että valo ei ehdi kulkea niiden välillä. Tällä tapaa pieni tasainen alue, jonka kaikki osat ovat aluksi kosketuksissa toisiinsa, pullistuu näkemäksemme maailmankaikkeudeksi.

Kazanasin työ unohtui vuosikymmeniksi – en tiedä miksi. Vieläkin inflaation isänä mainitaan usein Alan Guth, joka on pokannut siitä palkintojakin. Guthin artikkeli ilmestyi vuonna 1981, ja sen perusidea oli sama, minkä Kazanas oli jo esittänyt, vaikka Guthin käsittely olikin laajempi ja selkeämpi. Guth myös otti käyttöön termin inflaatio. Vuonna 1981 myös Katsuhiko Sato esitti saman idean Guthista riippumattomasti.

Joka tapauksessa osoittautui, että Kazanasin, Guthin ja Saton malli ei sellaisenaan sovi havaintoihin. Siinä varhaiset pienet alueet paisuvat kuin saippuakuplat, jotka inflaation loputtua törmäävät toisiinsa. Näiden törmäysten takia taivas näyttäisi enemmän vaahtomaiselta kuin tasaiselta. Mutta pian hahmotettiin, että laajeneminen on helppo saada kiihtymään hiukkasfysiikan kentillä muutenkin kuin tyhjön energiaa käyttämällä. Se onnistuu jopa Standardimallin Higgsillä, ilman tarvetta yhtenäisteorialle tai uusille kentille.

Tehosekoittimien kanssa puuhastelleet yleisen suhteellisuusteorian asiantuntijat eivät olleet hahmottaneet mahdollisuutta ongelman ratkaisemiseen kiihtyvällä laajenemisella. Idea ei lopulta ole monimutkainen, mutta monet suhteellisuusteoreetikot olivat omaksuneet sen ajatuksen, että aineella ei yksinkertaisesti voi olla sellaisia ominaisuuksia, jotka johtavat kiihtyvään laajenemiseen. Tarvittiin hiukkasfyysikoita toteamaan, että tunnettujen kenttien ominaisuudet itse asiassa ovat juuri sellaisia.

Jotkut suhteellisuusteoreetikot vastustivat inflaatiota vielä pitkään. Tämä on sinänsä huvittavaa, että inflaatio oli yksi hedelmällisimpiä asioita, mitä yleisen suhteellisuusteorian kentällä oli tapahtunut vuosikymmeniin. Inflaation vierastaminen johtui samasta syystä kuin sen hedelmällisyys: idea ylitti tieteenalan rajat tuomalla uusia ideoita hiukkasfysiikasta.

Lähes heti hahmotettiin, että inflaatio ei selitä vain sitä, miksi maailmankaikkeus näyttää suunnilleen samalta kaikkialla, vaan myös sen, mistä pienet erot johtuvat. Viatcheslav Mukhanov ja Gennady Chibisov laskivat vuonna 1981 Starobinskyn mallin puitteissa, miten kvanttivärähtelyt synnyttävät tyhjästä epätasaisuuksia, jotka voivat olla kaiken rakenteen siemeniä.

Näitä värähtelyjä tapahtuu niin aineessa kuin aika-avaruudessa, eli inflaatio yhdistää kvanttifysiikan ja yleisen suhteellisuusteorian. Inflaatio onkin ensimmäinen fysiikan alue, missä on tehty ennusteita kvanttigravitaatiosta ja onnistuneesti verrattu niitä havaintoihin. Koska alhaalta ylös lähdettäessä ei ole yleistä periaatetta, joka määräisi yksityiskohdat, on kuitenkin satoja erilaisia inflaatiomalleja.

Kunnianhimoiset syviin periaatteisiin pohjaavat ehdotukset kvanttigravitaatioteorioiksi kuten säieteoria ja silmukkakvanttigravitaatio eivät sen sijaan ole (ainakaan vielä) johtaneet valmiiseen teoriaan, joka tekisi ennusteita. Kvanttigravitaation saralla vaatimattomuus on ollut hyve. Säieteoria on tosin poikinut fysiikan ulkopuolella, kun sen kautta on löydetty matemaatikkoja kiinnostavia matemaattisia rakenteita.

Standardimallin kehittäminen oli kenties hiukkasfysiikan ja kosmologian saralla viimeinen tapaus, missä hienostuneiden teoreettisten rakenteiden kehittäminen menestyi, joskin se yhdistyi teorian kasaamiseen pala palalta kasaamisen. Suuri osa fysiikasta on teorioiden rakentamista tunnettujen asioiden tiimoilla, ja pieni osa on suurten ideoiden kehittelyä. Valtaosa yrityksistä kummallakaan saralla ei johda mihinkään. Viime aikoina ylös askel kerrallaan taapertava lähestymistapa on ollut menestyneempi, mutta on mahdoton sanoa, kumpi reitti vie seuraavaksi uuden äärelle.

26 kommenttia “Ylös pohjalta”

  1. Mistä inflaatio tiesi loppua eri alueissa samaan aikaan, jotta tuloksena oli homogeeninen maailmankaikkeus? Inflaatioperiodi kaiketi koostui kymmenistä e-kertaistumisista ja lopputuloksessa kuitenkin vain sadastuhannesosan tiheyserot.

    1. Syksy Räsänen sanoo:

      Inflaatiossa kentän potentiaalista aiheutuva voima ja maailmankaikkeudesta laajenemisesta aiheutuva kitka ovat tasapainossa, joten kiihtyvyys on suunnilleen nolla. Tämän takia kentällä ei ole muistia aiemmasta, tuleva kehitys riippuu vain sen nykyisestä arvosta. Tämä on kuin laskuvarjolla putoaisi.

      Inflaatio toisaalta pyyhkii pois klassisen fysiikan mukaiset epätasaisuudet kentän arvossa, jäljelle jää vain kvanttifluktuaatiot, jotka on tosiaan ovat 10^(-5) luokkaa.

    2. Oliko siis niin että inflaation aikana oli tilaparametrina olemassa vain inflatonikentän arvo, eli ei baryonista muttei myöskään pimeää ainetta, jotka molemmat syntyivät vasta inflatonin hajotessa hiukkasiksi? Jos näin on, niin saisiko tästä jotain rajoitteita pimeän aineen mahdollisille malleille?

      1. Syksy Räsänen sanoo:

        Mitä tahansa ainetta ennen inflaatiota olikin, kiihtyvä laajeneminen pian pudottaa kaikkien muiden paitsi skalaarikenttien energiatiheyden mitättömän pieneksi.

        Tosin ei tiedetä mitä ennen inflaatiota oli. Ei myös olla varmoja siitä, miten inflaatio alkoi.

      2. OK, tajusin nyt että kysymykseni oli tyhmä. Vaikka pimeän aineen tiheys onkin nykyään suurempi kuin baryonisen materian tiheys, molemmat olivat paljon pienempiä kuin kvarkki-antikvarkki-gluoniplasman tiheys inflaation loppuessa, koska melkein kaikki kvarkit ja antikvarkit annihiloivat myöhemmin toisensa, kuten myös niitä seuranneet hadronit ja leptonitkin. Eli fraasi että inflatoni hajosi standardimallin hiukkasiksi on kirjaimellisesti oikein, seassa oleva pimeä komponentti oli siihen nähden energiatiheysmielessä mitätön. (Korjaa jos tämä ei ole näin…tietysti jotain epävarmuuksia on kun ei tiedetä mitä pimeä aine on jne.)

        1. Syksy Räsänen sanoo:

          No, sitähän ei tiedetä miksi hiukkasiksi inflatoni tarkalleen hajosi, koska ei tiedetä mitä kaikkia hiukkasia on olemassa, eikä niiden hajoamisketjuja.

          On mahdollista, että inflatoni hajosi enimmäkseen tuntemattomiksi hiukkasiksi tai enimmäkseen tunnetuiksi.

  2. Ilkka Seittenranta sanoo:

    Ulkopuolisena tulee ihmeteltyä sitä että että aineen ylipäätään oletetaan olevan jotain pysyvää, voisi olettaa että aineen aivan pienin rakenneosanen on mahdoton havaita ja sen olemus joudutaan päättelemään – tässä vaiheessa fyysikot joutuvat perääntymään tavasta havaita ja osoittaa jotain.

    1. Syksy Räsänen sanoo:

      En ole varma siitä, mitä tarkoitat sillä, että aine on pysyvää. Useimmat hiukkaset ovat epästabiileja, ja stabiilitkin hiukkaset voivat tuhoutua.

      Sen mitä voi olla olemassa ja mitä voi havaita voi osoittaa vain tutkimus.

  3. Jyri T. sanoo:

    Mustan aukon säteily (Hawkingsin säteily) perustuu siihen, että musta aukko luo horisontin aika-avaruuteen. Oliko inflaation aikana vastaavaa säteilyä, kun laajeneminen synnytti samantyyppisen horisontin (tosin nurin päin)? Oliko sillä merkitystä energiatiheyden kannalta?

    1. Syksy Räsänen sanoo:

      Mainio kysymys, joka menee vähän kauas, mutta yritän kuitenkin vastata lyhyesti.

      Mustan aukon horisontti on (ilman Hawkingin säteilyä) ikuinen, eli tapahtumahorisontti.

      Inflaation aikainen horisontti ei ole ikuinen (koska inflaatio loppuu) eli se ei ole tapahtumahorisontti.

      Jos nykyinen kiihtyminen jatkuu loputtomiin, kyseessä on tapahtumahorisontti, ja siihen oletettavasti liittyy Hawkingin säteilyä. (Ei ole yksimielisyyttä siitä, liittyykö ei-ikuiseen horisonttiin tällaista säteilyä.)

      Joka tapauksessa kosmisen horisontin aiheuttama säteily inflaation aikana on mitättömän pientä verrattuna inflaation aiheuttavan kentän vaikutukseen. Jos nykyinen kiihtyvä laajeneminen jatkuisi ikuisesti, näin ei välttämättä ole, vaan horisontin Hawkingin säteily saattaa lopettaa sen. Tommi Markkanen (jonka kanssa minulla on joskus ollut yhteistyötä, ei tosin tästä) on tutkinut aihetta, mutta näitä asioita ei tiedetä varmasti.

  4. Lentotaidoton sanoo:

    ”Tommi Markkanen (jonka kanssa minulla on joskus ollut yhteistyötä, ei tosin tästä) on tutkinut aihetta, mutta näitä asioita ei tiedetä varmasti.”

    Sen verran mielenkiintoista että löytyiskö jostain luettavaa?

    1. Syksy Räsänen sanoo:

      Mitään populaaria esitystä siitä ei taida olla, tässä on Tommin artikkeli:

      https://arxiv.org/abs/1703.06898

  5. Onko niin että inflaatio ei varsinaisesti tarvitse mitään alkuehtoa (tiheyttä, laajenemisnopeutta…), vaan riittää kunhan Lagrange (SM+CDM mukaanlukien inflatoni-skalaarikentän potentiaalin muoto) on juuri oikein, niin tuloksena on meidän näköinen maailmankaikkeus? (Ja lambda mutta vain jos halutaan hifistellä.)

    1. Syksy Räsänen sanoo:

      Tämä on suurin avoin kysymys inflaatiossa. Kentän pitää olla tarpeeksi tasainen, jotta inflaatio käynnistyy. Mutta hyvin epätasaisista alueista ei välttämättä ole haittaa, jos ne romahtavat mustiksi aukoiksi: kunhan inflaatio alkaa jossain osassa avaruutta, sen tilavuus kasvaa niin paljon niin nopeasti, että pian käytännöllisesti katsoen koko maailmankaikkeuden tilavuus inflatoi.

      Inflaation alkamisen ehtoja on tutkittu analyyttisesti, ja ihan viime vuosina on tehty myös tietokonesimulaatioita yleisessä suhteellisuusteoriassa, mutta asiasta ei ole täyttä selvyyttä.

    2. Jos pimeä aine on pieniä mustia aukkoja, silloin kaiketi niitä on inflaation alkaessa pitänyt ollakin todella tiheässä eli varmaan enemmistö silloisesta energiatiheydestä? Vai dilutoiko inflaatio aukot yhettömiin kuten monopolitkin vaikka niitä olisi alussa kuinka tiheässä?

      1. Syksy Räsänen sanoo:

        Inflaatio pyyhkii mustat aukot pois siinä missä muutkin epätasaisuudet.

        Kaikki nykymaailmankaikkeiden mustat aukot ovat syntyneet inflaation jälkeen.

  6. Jos olisi hiukkaskiihdytin jolla pääsisi tutkimaan inflaatioepookin fysiikkaa, mitä silloin näkyisi? Voisiko esimerkiksi miljoonan teraelektronivoltin raskasionitörmäyksessä syntyä baby-inflaatio joka toimisi vähän aikaa? Tulisivatko inflaation reheating-tulokset tänne meidän maailmankaikkeuteemme vai muodostaisivatko ne oman kuplansa jota ei pysty havaitsemaan? Käsittääkseni inflaatio tavallaan luo energiaa tyhjästä, mutta havaitsisimmeko mekin tuollaisessa törmäyksessä energian syntymistä tyhjästä vai menisikö se energia jonnekin piiloon meiltä?

    Kysymys on toki spekulatiivinen koska tuollaisia kiihdyttimiä ei näy ihan nurkan takana ainakaan vielä, mutta 1e9 GeV törmäysenergia on satatuhatta LHC:tä joten kyllä sellainen aurinkokuntaan hyvin mahtuisi. Katsoin energiat sivulta https://en.wikipedia.org/wiki/Chronology_of_the_universe, jossa annetaan inflaatioperiodille 1e15..1e9 GeV.

    1. Syksy Räsänen sanoo:

      Inflaatio ei alkaisi uudelleen. Inflaatiota ajaa koherentti kenttä, joukko inflatonihiukkasia käyttäytyy eri tavalla. (Vrt. vakiosähkökenttä ja kaasu fotoneita.)

    2. Kiitos vastauksesta. Unohdin mainita että kosmisen säteilyn energiaennätys on niinkin iso kuin 3e11 GeV, eli ilman kiihdytintäkin sellaisia törmäyksiä tapahtuu.

      1. Syksy Räsänen sanoo:

        Oleellista on törmäyksen energia massakeskipistekoordinaatistossa, joka kosmisten säteiden törmäyksissä on korkeimmillaan samaa luokkaa kuin LHC:ssä.

  7. Jos ymmärsin oikein, inflaatio on poistanut jäljet sitä edeltäneestä menneisyydestä eikä prosessia voi tuottaa uudelleen laboratoriossa. Miten siis pitäisi edetä sen tutkimisessa?

    Analoginen dilemma on muuten elämän synnyn tutkimuksessa.

    1. Syksy Räsänen sanoo:

      Askel kerrallaan. Kosmologiset havainnot tarkentuvat koko ajan, ja niillä saadaan tarkemmin selvitettyä miten inflaatio tapahtui. On myös erilaisia teoreettisia ehdotuksia uudenlaisista jäljistä (joista osa kertoisi ajasta ennen inflaatiota) joita etsiä, ja niitä onkin etsitty. Mutta ei niistä tässä sen enempää!

  8. Onkohan seuraava oikein ajateltu:

    Eksponentiaalisen inflaation exp(H*t) aikana Heisenbergin energia-aika -relaation nojalla lämpötila T (eli energia) on vähintään H, koska 1/H on se relevantti aikaskaala eli laajenemisen e-folding -aika, jossa ilmiöitä tarkastellaan. Tämä nollapiste-energia tai ”Heisenbergin lämmitys” taistelee punasiirtymästä johtuvaa eksponentiaalista jäähtymistä vastaan, ja sitä kautta hidastaa inflaation päättymistä(?)

    Numeroarvoja: Jos inflaation aikana 1/H = 1e-35 s, ”Heisenberg-lämpötilaksi” saadaan 1e24 K, mikä ei liene merkityksettömän pieni. Wikipedian mukaan inflaation aikana lämpötila putosi 1e27 kelvinistä 1e22 kelviniin, eli tuo 1e24 K on suunnilleen puolivälissä tuota lämpötilahaarukkaa.

    Varmaankin tämä Heisenbergiin liittyvä fysiikka ja sen vaikutus inflaation kestoon on jotenkin leivottuna sisään niissä formulaatioissa joilla ammattilaiset käsittelevät inflaatioepookkia?

    1. Syksy Räsänen sanoo:

      Wikipedian fysiikan artikkelien luotettavuus vaihtelee paljon.

      Inflaation aikana tosiaan oletettavasti on näennäiseen horisonttiin liittyvä Hawkingin lämpötila H/(2pi). Inflaatiota ajavan kentän tyypillisest kvanttivärähtelyt ovat tuota samaa luokkaa. Kokonaisenergiatiheyteen tällä ei kuitenkaan ole vaikutusta, koska se on luokkaa H^2 M_Pl^2, missä M_Pl on Planckin manssa, ja M_Pl>>H.

      Merkintä on sen verta vanha, että ei tässä tästä varmaan sen enempää.

      1. Kiitos. Kun googlasin, huomasin että tässä Fairbairn-Markkanen-Rodriguez Roman (2018) paperissa (https://www.ncbi.nlm.nih.gov/pmc/articles/PMC6438654/ ) oli paljolti tehty se mitä heuristisesti ajoin takaa.

Vastaa

Sähköpostiosoitettasi ei julkaista. Pakolliset kentät on merkitty *